Respiratory Question

You might also like

Download as docx, pdf, or txt
Download as docx, pdf, or txt
You are on page 1of 79

Question #1

QID: 123
Topic: Diagnostic Investigations
Subject: Medicine

A 70-year-old man presents with recurrent bilateral lower lobe pneumonia with mild
dysphagia. On exam he is otherwise normal. What is the procedure of choice to arrive
at the diagnosis?  

a)  Barium swallow
b)  Serum protein electrophoresis
c)  CBC
d)  ESR

The correct answer is a)

Explanation:
Swallowing disorders are common, especially in the elderly, and may cause
dehydration, weight loss, aspiration and airway obstruction.

Aspiration is the passage of food or liquid through the vocal folds. Persons who
aspirate are at increased risk for the occurrence of serious respiratory sequelae,
including airway obstruction and aspiration pneumonia.

The most valuable investigations in patients with suspected esophageal dysphagia


include a barium swallow study, endoscopy and esophageal manometry. Diagnostic
study of choice would be a barium swallow. The barium will show any structural
defects such as strictures, narrowing or tumors.

Question #2
QID: 1136
Topic: Diagnostic Investigations
Subject: Medicine

Which of the following pulmonary function tests most reliably discriminates “pure”
chronic bronchitis from emphysema?

a)  Total lung capacity


b)  Functional residual capacity
c)  Residual volume
d)  Single breath diffusing capacity
e)  Flow at 50% vital capacity

The correct answer is d)

Explanation:
The single breath diffusing capacity, or DLCO, is a measure of the ability of the lungs
to diffuse oxygen into, and carbon dioxide from, the bloodstream. Measurement of
diffusing capacity of the lung for carbon monoxide (DLCO) should be considered a
routine initial test in evaluation of chronic airflow obstruction, particularly in
advanced disease. DLCO has been established as sensitive in detection of emphysema
that is associated with loss of alveolar surface area and pulmonary circulation.

Question #3
QID: 3333
Topic: Diagnostic Investigations
Subject: Medicine

A 60-year-old white male comes to your office for evaluation of a chronic cough
productive of large amounts of sputum, accompanied by dyspnea on exertion. He has
smoked 2 packs of cigarettes a day for the past 40 years.

The best diagnostic test for evaluating this problem is:  

a)  arterial blood gases


b)  alpha1-antitrypsin
c)  brain natriuretic peptide (BNP)
d)  CT of the chest
e)  spirometry

The correct answer is e)

Explanation:
Considering the patient's history, the most likely diagnosis is COPD. It is important to
note that in questions that ask the single best answer, when two choices seem to be a
possibility, then the most common is the right answer.

In patients with suspected COPD, the best diagnostic test is office spirometry (choice
E). If the FEV1/FVC ratio is <70% and the FEV1 is <80% of predicted, the patient has
COPD. This generally occurs in mid- to late life. While cigarette smoking is the
largest single risk factor, only 20% of smokers develop clinically significant COPD.
The second most common risk factor is alpha1-antitrypsin deficiency, which causes
1% of cases. These patients present with cough, sputum production, and dyspnea on
exertion. They often experience orthopnea soon after reclining, whereas patients with
heart failure typically experience orthopnea several hours after reclining, when fluid
mobilizes from the lower extremities.

>Arterial blood gases (choice A) would be the best test to evaluate the severity of an
acute attack or exacerbation.
>alpha1-antitrypsin(choice B) would be useful in patients with alpha1-antitrypsin
deficiency. These patients develop COPD early even without exposure to tobacco
products.
>brain natriuretic peptide (choice C) is useful in the assessment of patients with
congestive heart failure. The most likely diagnosis in this patient is COPD.
>CT of the chest (choice D) may be useful in the diagnosis of emphysema.
However,It would only  be used as an adjunctive study in a patient suspected of
COPD.

Question #4
QID: 5586
Topic: Diagnostic Investigations
Subject: Medicine

A 42-year-old female presents with a 2-day history of chest pain. She describes the
pain as sharp, located in the right upper chest, and worsened by deep breathing or
coughing. She also complains of shortness of breath. She was previously healthy and
has no recent history of travel. Her vital signs are normal. A pleural friction rub is
noted on auscultation of the lungs. The remainder of the examination is normal. An
EKG, cardiac enzymes, oxygen saturation, and a D-dimer level are all normal.

Which one of the following would be most appropriate at this point?


a)  No further testing
b)  A chest radiograph
c)  An antinuclear antibody test
d)  Echocardiography
e)  Pulmonary angiography

The correct answer is b)

Explanation:
This patient has pleurisy. Patients presenting with pleuritic chest pain may have life-
threatening disorders, and pulmonary embolism, acute myocardial infarction, and
pneumothorax should be excluded. While 5%-20% of patients with pulmonary
embolism present with pleuritic chest pain, this patient has no risks for pulmonary
embolism and the normal D-dimer level obviates the need for further evaluation.
Moderate- to high-risk patients may need a helical CT scan or other diagnostic testing.

An EKG and chest radiograph are recommended in the evaluation of acute/subacute


pleuritic chest pain. The chest radiograph will exclude pneumothorax, pleural
effusion, or pneumonia. An echocardiogram would not be indicated if the cardiac
examination and EKG are normal. An antinuclear antibody level could be considered
in recurrent pleurisy or if other symptoms or signs of lupus were present, but it would
not be indicated in this patient.

Most cases of acute pleurisy are viral and should be treated with NSAIDs unless the
workup indicates another problem.
Question #5
QID: 178
Topic: Tuberculosis
Subject: Medicine

A patient with history of occupational exposure (worked in a silica mine) 20 years


back, now presents with 6 month history of cough and weight loss. CXR shows
infiltrates and fibrosis in the upper lobes.

What is the most likely diagnosis?  

a)  Pneumonia
b)  Tuberculosis
c)  Mesothelioma
d)  Rapidly progressive silicosis

The correct answer is b)

Explanation:
Pulmonary tuberculosis is associated with silica dust exposure as an occupational
disease. It is caused by Mycobacterium tuberculosis in employees who have been
exposed to crystalline silica dust in the workplace.

Symptoms include cough, hemoptysis, fever, chills and weight loss. Diagnosis is
made by sputum cultures and CXR. Treatment is usually with a combination of
medicines such as Isoniazid, Rifampin, Ethambutol and Pyrazinamide.

> Pneumonia would not be this long of a duration and would not present with weight
loss.
> Mesothelioma usually affects the lower lobes with pleural thickening.
> Rapidly progressive silicosis develops within 6 months of acute exposure.

Question #6
QID: 2901
Topic: Tuberculosis
Subject: Medicine

A 55-year-old patient with a known history of chronic alcohol abuse presents to your
office because he has been losing weight, he always feels tired and he has been
coughing. A physical examination reveals a temperature of 38.8˚C (101.8˚F).
Hepatomegaly is noted on abdominal palpation. A chest radiograph shows diffuse,
well-defined nodules that are less than 5 mm in diameter. Alfa-fetoprotein is within
normal limit.

Which one of the following is the most likely diagnosis?


a)  Bronchiectasis 
b)  Reactivation pulmonary tuberculosis 
c)  Primary pulmonary tuberculosis 
d)  Miliary tuberculosis 
e)   Hepatic carcinoma

The correct answer is d)

Explanation:
Diffuse small pulmonary nodules are characteristic of miliary tuberculosis (TB). This
disease is seen commonly in those who are immunosuppressed due to alcohol abuse.
This patient's symptoms of weight loss, fatigue, and cough are very common in these
patients. The findings of nodules on chest X-ray and hepatomegaly suggest the
patient's TB may have muliple organs involvement. This is why miliary TB (choice
D) is the best choice.

> Bronchiectasis (choice A) may be a complication of endobronchial TB, but occurs


late in the course
> Reactivation pulmonary TB (choice B)  may also be a complication and the
radiograph typically shows localized lesions. 
> Primary TB (choice C) usually consists of hilar adenopathy and a focal infiltrate
(Ghon complex). Primary TB is usually a mild process, but if it becomes severe it
usually presents with extensive lobar consolidation rather than diffuse nodules.
> Hepatic carcinoma (choice E) could be a complication of chronic  viral hepatitis or
chronic alcohol abuse, however, in the majority of patients it is preceded by cirrhosis
and the fact alfa-fetoprotein is negative in this patient should steer us away from this
choice.

Question #7
QID: 268
Topic: Pneumothorax
Subject: Medicine

A 20-year-old white man was stabbed in the chest in a bar fight and arrived at the
emergency room within 30 minutes. You noticed that the trachea is deviated away
from the side of the chest that suffered the puncture.

If you suspected pneumothorax, which one of the following would you expect to find
on the traumatized side?  

a)  Increased fremitus
b)  Increased breath sounds
c)  Dullness to percussion
d)  Hyperresonant percussion
e)  Wheezing and stridor

The correct answer is d)


Explanation:
A pneumothorax refers to a collection of gas in the pleural space resulting in collapse
of the lung on the affected side. A tension pneumothorax is a life-threatening
condition caused by air within the pleural space that is under pressure; displacing
mediastinal structures and compromising cardiopulmonary function. A traumatic
pneumothorax results from blunt or penetrating injury that disrupts the parietal or
visceral pleura.

Physical findings classically consist of absent tactile fremitus, hyperresonance to


percussion, and decreased breath sounds on the side with the pneumothorax. If the
pneumothorax is large, the side with the pneumothorax may be enlarged with the
trachea visibly shifted to the opposite side.

Diagnosis is made with an upright inspiratory chest x-ray. Treatment involves needle
thoracentesis to release the trapped air followed by placement of a chest tube.

Question #8
QID: 1531
Topic: Pneumothorax
Subject: Medicine

Tension pneumothorax is best diagnosed with:  

a)  Stat CT scan
b)  Chest x-ray
c)  Watch and wait
d)  Clinical exam
e)  None of the above

The correct answer is d)

Explanation:
Tension pneumothorax is a pneumothorax causing progressive rise in intrapleural
pressure to levels that become positive throughout the respiratory cycle and collapse
the lung, shift the mediastinum, and impair venous return to the heart. Air continues
to get into the pleural space but cannot exit. Without proper treatment, the impaired
venous return can cause systemic hypotension and respiratory and cardiac arrest
within minutes.

Tension pneumothorax most commonly occurs in patients receiving positive-pressure


mechanical ventilation (particularly during resuscitation). It is rarely a complication
of traumatic pneumothorax, when a chest wound acts as a one-way valve that traps
increasing volumes of air in the pleural space with inspiration. 

Tension pneumothorax is diagnosed clinically with hypertympany on the side of the


lesion, deviation of the trachea away from the lesion, and decreased breath sounds on
the affected side. There is usually associated elevated jugular venous pressure in the
neck veins. This is a clinical diagnosis, and once made an immediate needle
thoracostomy or tube thoracostomy should be performed to relieve the tension
pneumothorax.

Question #9
QID: 1695
Topic: Pneumothorax
Subject: Medicine

A 35-year-old male consults you about a vague chest pain he developed while sitting
at his desk earlier in the day. The pain is right-sided and was sharp for a brief time
when it began, but it rapidly subsided. There was no hemoptysis and the pain does not
seem pleuritic. His physical examination, EKG, and oxygen saturation are
unremarkable. A chest film shows a 10% right pneumothorax.

Which one of the following is true in this situation?  

a)  He is likely to be an overweight smoker with a chronic cough


b)  Rupture of subpleural bullae would be an unlikely cause of his problem
c)  Outpatient observation with a repeat chest radiograph in 24 hours is indicated
d)  A chest tube should be placed expeditiously
e)  After treatment his probability of recurrence is less than 15%

The correct answer is c)

Explanation:
The majority of patients presenting with spontaneous pneumothorax are tall, thin
individuals under 40 years of age. Most do not have clinically apparent lung disease,
and the chest pain is sometimes minimal at onset and may resolve within 24 hours
even if untreated. Patients with small pneumothorax involving less than 15% of the
hemithorax may have a normal physical examination, although tachycardia is
occasionally noted. The diagnosis is confirmed by chest radiographs. Studies of
recurrence have found that an average of 30% of patients will have a recurrence
within 6 months to 2 years. The treatment of an initial pneumothorax of less than 20%
may be monitored if a patient has few symptoms. Follow-up should include a chest
radiograph to assess stability at 24-48 hours. Indications for treatment include
progression, delayed expansion, or the development of symptoms. The majority of
patients with spontaneous pneumothoraces, and perhaps almost all of them, will have
subpleural bullae on a CT scan.

Question #10
QID: 5385
Topic: Pneumothorax
Subject: Medicine

A 40 year old male developed Adult Respiratory Distress Syndrome after a severe
attack of pancreatitis. He was admitted to the ICU, intubated and ventilated with
PEEP (Positive end-expiratory pressure) of 14 cm water, and FiO2 of 70%.

Suddenly, his pulse increases from 90 to 150/min; systolic Blood Pressure drops from
120 to 60 mm Hg; Central Venous Pressure increases from 20 to 50cm water.

Physical exam shows tracheal deviation to the left side and absent breath sounds on
the right side. JVD is noted.

What is the most appropriate next step in management?


a)  Chest tube
b)  Chest X ray
c)  EKG, CPK MB and Troponin
d)  IV heparin
e)  Needle thoracostomy

The correct answer is e)

Explanation:
Tension pneumothorax is not an uncommon complication of mechanical ventilation
when high PEEP is used. This occurs due to barotrauma.

Tracheal deviation + absent breath sounds + hypotension and JVD = Tension


pneumothorax.

The patient is dying. There is no time for diagnostic tests. Needle thoracentesis is life
saving.
A chest tube placement is appropriate after needle decompression of the pleural space.

Question #11
QID: 5559
Topic: Pneumothorax
Subject: Medicine

A 25-year-old male comes to the emergency department with the sudden onset of
moderate to severe right-sided chest pain and mild dyspnea. Vital signs are normal. A
chest film shows a loss of markings along the right lung margins, involving about
10%-15% of the lung space. The mediastinum has not shifted. The best INITIAL
treatment would be:
a)  strict bed rest
b)  oxygen supplementation and close observation
c)  decompression of the chest by insertion of a large-bore intravenous catheter into
the right second intercostal space at the midclavicular line
d)  immediate chest tube insertion using a water seal
e)  thoracotomy for wedge resection of pulmonary blebs

The correct answer is b)

Explanation:
A small spontaneous pneumothorax involving less than 15%-20% of lung volume can
be managed by administering oxygen and observing the patient. The pneumothorax
will usually resorb in about 10 days if no ongoing air leak is present. Oxygen lowers
the pressure gradient for nitrogen and favors transfer of gas from the pleural space to
the capillaries. Decompression with anterior placement of an intravenous catheter is
usually reserved for tension pneumothorax. Chest tube placement is used if
observation is not successful or for larger pneumothoraces. Strict bed rest is not
indicated.

Question #12
QID: 10879
Topic: Pneumothorax
Subject: Medicine

A 69-year-old male presents to the emergency department with complaints of


suddenly increased shortness of breath. He was diagnosed with COPD 10 years ago.
His medications are tiotropium, formoterol, and albuterol. Usually when he is short of
breath he uses albuterol, which alleviates the symptoms. However, "it has not helped
much this time." He has smoked 40 packs of cigarettes a year for the last 45 years.
On physical examination he has hyperresonance to percussion and decreased breath
sounds on auscultation at the right side. His oxygen saturation is 87%. Which of the
following is the most appropriate next step in management?
a)  Albuterol inhaler, ipratropium, and oral prednisone
b)  Sputum cultures and initiation of azithromycin
c)  Order chest radiograph
d)  Intubation and mechanical ventilation with 100% oxygen
e)  Oxygen administration at 3L/min nasal cannula

The correct answer is e)

Explanation:
This patient’s history of smoking and COPD, the presentation with shortness of
breath, decreased breath sounds on the right, and hyperresonance to percussion point
to a diagnosis of spontaneous pneumothorax. COPD is the most common cause of
secondary spontaneous pneumothorax accounting for 60% of cases on average.
Rupture of apical blebs is the usual cause. Out of the choices given, oxygen
administration at 3L/min nasal cannula (choice E) is the most appropriate initial step
in management of this patient. Oxygen treats hypoxemia and is associated with a 4-
fold increase in the rate of pleural air absorption compared with room air alone. If the
patient doesn't respond to oxygen supplementation, chest tube placement would be the
most appropriate next-in-line treatment.

> Albuterol inhaler, ipratropium, and oral prednisone (choice A) would be appropriate
for the management of COPD exacerbation.
> Sputum cultures and initiation of azithromycin (choice B) would be appropriate if
community acquired pneumonia is suspected. This patient’s clinical scenario suggests
spontaneous pneumothorax.
> Order chest radiograph (choice C) should be part of the management plan of this
patient’s condition, but oxygen should be given prior to imaging investigations.
> Intubation and mechanical ventilation with 100% oxygen (choice D) is incorrect.
The appropriate initial oxygen supplementation should be done by nasal cannula.

Key point:
Sudden increase of shortness of breath, unilateral decreased breath sounds on
auscultation and hyperresonance to percussion in a patient with COPD history is
suggestive of spontaneous pneumothorax. Oxygen supplementation (through a nasal
cannula) is an appropriate initial treatment of spontaneous pneumothorax.

Question #13
QID: 261
Topic: Pulmonary Embolism
Subject: Medicine

An elderly man traveled to North America from Australia. After 5 days he develops
dyspnea, chest pain, tachycardia and is rushed to the hospital.

What is the most likely diagnosis?  


a)  Pulmonary embolism
b)  Pneumonia
c)  Myocardial infarction
d)  Aortic dissection

The correct answer is a)

Explanation:
Pulmonary embolism (PE) is the occlusion of one or more pulmonary arteries by
thrombi that originate elsewhere, typically in the large veins of the lower extremities
or pelvis. Nearly all PEs arise from thrombi in the lower extremity or pelvic veins
(deep venous thrombosis (DVT).

'Virchow's triad' of stasis, hypercoagulability and endotheloal damage are risk factors
for developing DVT.

This man has sat on a very long airplane flight from Australia to North America.
Meeting the criteria for stasis. His symptoms are classic for PE which can include:
acute dyspnea, pleuritic chest pain, tachycardia and tachypnea. The first symptom in
an older patient may be altered mental status.

Question #14
QID: 1192
Topic: Pulmonary Embolism
Subject: Medicine

When considering a diagnosis of acute symptomatic pulmonary embolism, which of


the following diagnostic modalities would be the best to detect it? 

a)  Computed tomography angiography


b)  Ventilation-perfusion lung scan
c)  Bilateral leg venograms
d)  Arterial blood gas analysis
e)   D-dimer

The correct answer is a)

Explanation:
Pulmonary embolism is the occlusion of one or more pulmonary arteries by thrombi
that originate elsewhere, typically in the large veins of the lower extremities or pelvis.
Risk factors are conditions that impair venous return, conditions that cause endothelial
injury or dysfunction, and underlying hypercoagulable states. Symptoms are
nonspecific and include dyspnea, pleuritic chest pain, cough, and, in severe cases,
syncope or cardiorespiratory arrest. Signs are also nonspecific and may include
tachypnea, tachycardia, hypotension, and a loud pulmonic component of the 2nd heart
sound.  CT angiography (choice A) is considered standard in detecting pulmonary
embolism.

> Ventilation-perfusion lung scan (choice B) is a useful screening test, but it is not
very specific.
> Bilateral leg venograms (choice C) would be helpful in a patient suspected of deep
vein thrombosis. This is a common precursor of pulmonary embolism but it is not the
best study to detect a pulmonary embolism that has already taken place.
> Arterial blood gas analysis (choice D) may reveal hypoxemia, hypocapnia, and
respiratory alkalosis; however, the predictive value of hypoxemia is quite low.
> D-dimer (choice E) is done in patients with low probability of pulmonary embolism
based on clinical presentation.

Question #15
QID: 1275
Topic: Pulmonary Embolism
Subject: Medicine

A 44 year old man sustained a comminuted fracture of his left tibia and fibula 4
months ago. For the past 3 months he has been in the rehabilitation unit with his leg
fully immobilized. Three hours ago he suddenly developed chest pain and shortness
of breath, and he has just been brought to the emergency department for further
evaluation. On examination he describes an aching discomfort over the right superior
anterior chest and the right scapula posteriorly. The family history is strongly positive
for heart disease.

The presence of a right pleural friction rub in this patient would suggest which of the
following?  

a)  Pericarditis
b)  Pneumonia
c)  Pneumothorax
d)  Pulmonary embolus with infarction
e)  Pulmonary embolus without infarction

The correct answer is d)

Explanation:
Pain and pleural frictional rub is almost diagnostic of infarction of the overlying
pleura.

>Pericardial rub due to pericarditis is localized to the lower left side of the heart; there
is no evidence for this diagnosis here.
>Pneumonia, which is another complication of immobility, may present with similar
signs and symptoms, but should be accompanied with fever and leukocytosis.
>A pneumothorax and a pulmonary embolus without infarction do not typically
present with a pleural friction rub.
Question #16
QID: 1549
Topic: Pulmonary Embolism
Subject: Medicine

Contraindications to thrombolytic therapy in treatment of pulmonary embolism


include which one of the following?  

a)  Age > 80
b)  Intracranial neoplasms
c)  Blood pressure of 140/85
d)  GI bleeding 6 months ago

The correct answer is b)

Explanation:
Contraindications to thrombolytic therapy include eye or central nervous system
surgery within the preceding 2 weeks, intracranial neoplasms or vascular
abnormalities, stroke within the preceding 2 months, active bleeding, severe
hypertension, and allergy to thrombolytic agents. Age is not a consideration.

Question #17
QID: 3211
Topic: Pulmonary Embolism
Subject: Medicine

A 45-year-old female presents to an urgent care center complaining of left-sided chest


pain for the past 2 days. The pain is nonradiating and sharp in character, and increases
with deep inspiration. She has no associated shortness of breath, cough, nausea,
diaphoresis, or dizziness. She has no significant past medical history or recent travel
history.

On examination she is afebrile, with a pulse rate of 92 beats/min, a blood pressure of


116/72 mm Hg, and a respiratory rate of 12/min. Her lungs are clear and her heartbeat
is regular with no murmurs. Her lower extremities have no edema, tenderness, or
varicosities.

Which one of the following is the most appropriate next step in her evaluation?  

a)  A high-sensitivity D-dimer test


b)  A troponin I level
c)  Ultrasound examination of the veins of the lower extremities
d)  A multidetector helical CT of the chest
e)  An antinuclear antibody (ANA) level
The correct answer is a)

Explanation:
This patient has a low pretest probability of pulmonary embolism based on the Wells
criteria. She would be a good candidate for a high-sensitivity D-dimer test, with a
negative test indicating a low probability of venous thromboembolism.

> In patients with a low pretest probability of venous thromboembolism, an


ultrasound or helical CT would not be the recommended initial evaluation.
> Neither troponin I nor an ANA level would be part of the recommended initial
evaluation.

Question #18
QID: 5416
Topic: Pulmonary Embolism
Subject: Medicine

A 60-year-old female presents to the emergency department complaining of shortness


of breath. She arrived from Australia yesterday after a long visit at her daughter’s. She
has developed swelling of her left leg yesterday evening followed by shortness of
breath that began today. Her past medical history is unremarkable.
Vital signs are significant for pulse of 120/min and a RR of 24/min. Physical exam is
non contributory except for the swollen tender left leg. Chest x-ray is normal.
Which of the following arterial blood gas test findings would suggest pulmonary
embolism?  

a)  Decreased A-a gradient


b)  Decreased PaCO2
c)  Increased PaCO2
d)  Normal PaO2
e)  pH less than 7.38

The correct answer is b)

Explanation:
The patient is most likely having pulmonary embolism. The shortness of breath that
follows the leg swelling with the associated tachycardia and tachypnea suggests the
diagnosis of PE. A low PaCO2 (less than 40mmHg) is usually seen due to the
associated tachypnea.

> The A-a gradient (choice A) is usually increased due to the ventilation/perfusion
mismatch.
> PaO2 (choice D) is almost always decreased in PE because of the low perfusion i.e.
less oxygenation.
> Respiratory alkalosis and not acidosis (choice E) is usually seen in PE.
Question #19
QID: 6037
Topic: Pulmonary Embolism
Subject: Medicine

A 42-year-old female presents to the emergency department with pleuritic chest pain.
Her probability of pulmonary embolism is determined to be low.

Which one of the following should be ordered to further evaluate this patient?
a)  Brain natriuretic peptide (BNP)
b)  CT pulmonary angiography
c)  ELISA-based D-dimer
d)  A cardiac troponin level
e)  A ventilation-perfusion lung scan

The correct answer is c)

Explanation:
Patients who have a low or moderate pretest probability of pulmonary embolism
should have d-dimer testing as the next step in establishing a diagnosis.

Question #20
QID: 308
Topic: Idiopathic Pulmonary Fibrosis
Subject: Medicine

A 51-year-old man complains of a 6 month history of shortness of breath and cough.


Physical examination reveals hypertension, digital clubbing and fine bibasilar
inspiratory crackles.

Which one of the following is the most likely diagnosis?  

a)  Asthma
b)  Reflux-induced cough
c)  Chronic bronchitis
d)  Idiopathic pulmonary fibrosis
e)  Sarcoidosis

The correct answer is d)

Explanation:
Idiopathic pulmonary fibrosis (IPF), also known as cryptogenic fibrosing alveolitis, is
a chronic, progressive interstitial lung disease with an unknown cause. It is one of the
two classic interstitial lung diseases, the other being sarcoidosis.

IPF is slightly more common in males and usually presents in patients greater than 50
years of age. Average survival from time of diagnosis varies between 2.5 and 3.5
years, depending on severity, although some patients live greater than 10 years.

Symptoms are gradual in onset. The most common are dyspnea (difficulty breathing),
but also include nonproductive cough, clubbing (a disfigurement of the fingers), and
fine bibasilar inspiratory crackles (Velcro crackles).

> Sarcoidosis is an immune system disorder characterized by non-caseating


granulomas (small inflammatory nodules). It most commonly arises in young adults.
The cause of the disease is still unknown. Virtually any organ can be affected;
however, granulomas most often appear in the lungs or the lymph nodes. Common
symptoms are vague, such as fatigue unchanged by sleep, lack of energy, weight loss,
aches and pains, arthralgia, dry eyes, blurry vision, shortness of breath, a dry hacking
cough or skin lesions such as erythema nodosum.

Question #21
QID: 369
Topic: Bronchiectasis
Subject: Medicine

A 30-year-old man has a history of recurrent pneumonias and chronic cough dating
from early childhood. The cough, which is worse in the morning and on lying down,
is productive of foul-smelling purulent sputum which is occasionally bloody-tinged.
The patient is chronically ill and has clubbed fingers. Rales are heard over the
posterior lung bases.

Which one of the following is the most likely diagnosis?  

a)  Chronic bronchitis
b)  Pulmonary aspergillosis
c)  Pulmonary neoplasm
d)  Chronic obstructive emphysema
e)  Bronchiectasis

The correct answer is e)

Explanation:
Bronchiectasis is dilation and destruction of larger bronchi caused by chronic
infection and inflammation. Common causes are cystic fibrosis, immune defects, and
infections, though some cases appear to be idiopathic. Symptoms are chronic cough
and purulent sputum expectoration; some patients may also have fever and dyspnea.

Diagnosis is based on history and imaging, usually involving high-resolution CT,


though standard chest x-rays may be diagnostic. Treatment and prevention of acute
exacerbations are with antibiotics, drainage of secretions, and management of
complications, such as superinfection and hemoptysis. Treatment of underlying
causes is important whenever possible.

Question #22
QID: 3146
Topic: Bronchiectasis
Subject: Medicine

You see a 55 year old female for the first time. She has a 2-year history of chronic
daily cough; thick, malodorous sputum; and occasional hemoptysis. She has been
treated with antibiotics for recurrent respiratory infections, but is frustrated with her
continued symptoms. She has never smoked. Her FEV1 :FVC ratio is 60% and a CT
scan shows bronchial wall thickening and luminal dilation.

The most likely diagnosis is:  

a)  Emphysema
b)  Bronchiectasis
c)  Chronic bronchitis
d)  Bronchiolitis
e)  Asthma

The correct answer is b)

Explanation:
Bronchiectasis is an illness of the bronchi and bronchioles involving obstructive and
infectious processes that injure airways and cause luminal dilation. In addition to
daily viscid, often purulent sputum production with occasional hemoptysis, wheezing
and dyspnea occur in 75% of patients. Emphysema and chronic 1 bronchitis, forms of
COPD, also cause a decreased FEV :FVC ratio, but the baseline sputum is generally
mucoid and luminal dilatation of bronchi is not characteristically present.
Bronchiolitis is usually secondary to respiratory syncytial virus infection in young
children. Asthma is not characterized by the sputum and CT findings seen in this
patient.

Question #23
QID: 5384
Topic: Bronchiectasis
Subject: Medicine

A 60 year old male presents to the office complaining of a productive cough for the
past 3 months.
He mentions that the sputum is plentiful and foul smelling. He is a chain smoker.
Vitals are within normal limits except for a mild fever of 37.60C. Physical exam
shows finger clubbing. Auscultation of the lungs shows coarse crepitations in both
lungs.

Chest x-ray reveals increased vascular markings and peribronchial thickening.

What is the diagnostic test of choice for this patient?


a)  Bronchography
b)  Bronchoscopy
c)  High resolution CT Scan of the lung
d)  Sputum for Acid Fast bacilli
e)  Sputum gram stain and culture

The correct answer is c)

Explanation:
High resolution CT scan of the lung is the diagnostic modality of choice for
bronchiectasis. Copious foul smelling sputum is a clue. The chest x-ray of our patient
is characteristic of bronchiectasis: peribronchial thickening (Tram track appearance).

Bronchography is an old method of diagnosis and it has been replaced nowadays by


HRCT.

Bronchoscopy and biopsy would be warranted if a bronchial lesion were seen on chest
X ray or CT scan.

Sputum examination & Culture and sputum smear for AFB should be done in clinical
practice; however the question is clear and asks for the diagnostic modality of choice.

Question #24
QID: 432
Topic: PFT
Subject: Medicine

You are performing pulmonary function tests (PFT) on a 80 year old male with
persistent cough. Which one of the following PFT results would occur normally with
age?  

a)  Decrease in airway dimensions


b)  Fall in FEV1/FVC
c)  Fall in arterial pO2
d)  Increase in arterial pCO2
e)  Decrease in arterial pH

The correct answer is b)


Explanation:
Pulmonary function tests (PFT’s) provide measures of flow rates, lung volumes, gas
exchange, and respiratory muscle function. Basic pulmonary function tests available
in the ambulatory setting include spirometry and pulse oximetry; these tests provide
physiologic measures of pulmonary function and can be used to quickly narrow a
differential diagnosis and suggest a subsequent strategy of additional testing or
therapy.

FEV1, FVC, FEF25-75% and PEF increase and FEV1/FVC% decrease with age until
about 20 years old in females and 25 years in males. After this, all indices gradually
fall, although the precise rate of decline is probably masked due to the complex
interrelationship between age and height. The fall in FEV1/FVC% with age in adults
is due to the greater decline in FEV1 than FVC.

Note: Pulmonary function tests do not measure airway dimensions.

Question #25
QID: 2059
Topic: PFT
Subject: Medicine

In the elderly, which one of the following measures of pulmonary function is


characteristically increased?  

a)  Total lung capacity


b)  Vital capacity
c)  Resting PaO2 arterial tension
d)  Functional residual capacity
e)  One-second forced expiratory volume (FEV1)

The correct answer is d)

Explanation:
Although vital capacity declines with age, total lung capacity remains constant. The
reduction in vital capacity results from an increase in residual volume. This increase
in residual volume and functional residual capacity results from the collapse of small
airways that occurs at higher lung volumes as age increases. Residual volume
increases nearly 50% between early adulthood and age 70. In addition, standard
spirometric measurements of lung function (i.e., forced expiratory volume in 1
second, peak expiratory flow rate, and maximal expiratory flow volume) have been
shown to decline with age. Arterial oxygen tension also slowly declines with age.

Question #26
QID: 5502
Topic: PFT
Subject: Medicine

A 67 year old smoker with a history of pulmonary tuberculosis at 22 years of age


presents with a 6 month history of increasing shortness of breath. On office
spirometry his FVC is 60% of predicted, his FEV1 is 80% of predicted, and his
FEV1 / FVC ratio is 0.8.

What is the most appropriate next step?


a)  Refer to a pulmonary laboratory for static lung volume measurement and diffusion
studies
b)  Perform a bronchodilator challenge test
c)  Investigate for nonpulmonary causes for the patient’s symptoms
d)  Order a PPD skin test
e)   Prescribe albuterol

The correct answer is a)

Explanation:
Pulmonary function tests are usually classified as normal, compatible with a
restrictive defect, or consistent with obstructive airway disease. In restrictive
ventilatory processes, the FVC is decreased, the FEV1 is decreased or normal, and the
absolute FEV1 /FVC is >0.7. In obstructive airway problems, findings include a
normal or decreased FVC, a decreased FEV1 , and an absolute FEV1 /FVC <0.7.

When simple spirometry suggests a restrictive ventilatory problem, the patient should
undergo full pulmonary function testing for static lung volume measurements and
diffusing capacity of the lung for carbon monoxide. If spirometry suggests an
obstructive problem, it should be repeated after administering an inhaled
bronchodilator. Because this patient has a history of pulmonary tuberculosis, a TB
skin test is inappropriate and would not provide any useful information.

Question #27
QID: 434
Topic: COPD
Subject: Medicine

A 56-year-old patient presents with polycythemia. He is found to have low oxygen


saturation, increased red blood cells mass, normal plasma volume and increased
erythropoietin levels.

Which one of the following is the most likely diagnosis?  

a)  Polycythemia rubra vera


b)  Chronic obstructive pulmonary disease (COPD)
c)  Stress polycythemia
d)  Renal adenocarcinoma
e)  Congenital spherocytosis

The correct answer is b)

Explanation:
Chronic obstructive pulmonary disease (COPD) is persistent obstruction of the
airways occurring with emphysema, chronic bronchitis, or both disorders.

In the earlier stages of COPD, oxygen levels in the blood may be decreased, but
carbon dioxide levels remain normal. In the later stages, carbon dioxide levels
increase and oxygen levels fall.

The decrease in oxygen levels in the blood stimulates the bone marrow to send more
red blood cells into the bloodstream, a condition known as secondary polycythemia.
The decrease in oxygen levels in the blood also increases the pressure in the artery
through which blood flows from the heart to the lungs (pulmonary artery).

Question #28
QID: 1198
Topic: COPD
Subject: Medicine

Each of the following is a correct statement about COPD, except:  

a)  The type of emphysema associated with smoking is usually centriacinar


b)  Clubbing is a typical clinical feature of COPD
c)  Inhaled corticosteroids are not recommended as monotherapy for COPD
d)  Smoking cessation is the single most effective therapy for the majority of COPD
patients
e)  Long-term oxygen therapy improves survival in hypoxemic patients with COPD

The correct answer is b)

Explanation:
Chronic obstructive pulmonary disease is partially reversible airflow obstruction
caused by an inflammatory response to inhaled toxins, often cigarette smoke. α1-
Antitrypsin deficiency and various of occupational exposures are less common causes
in nonsmokers. Symptoms are productive cough and dyspnea that develop over years;
common signs include decreased breath sounds, prolonged expiratory phase of
respiration, and wheezing.

> Centriacinar emphysema (choice A) begins in the respiratory bronchioles and


spreads peripherally. Also termed centrilobular emphysema, this form is associated
with long-standing cigarette smoking and predominantly involves the upper half of
the lungs.
> The use of systemic steroids in the treatment of acute exacerbations is widely
accepted and recommended, given their high efficacy. However, Inhaled
corticosteroids are not recommended as monotherapy and should be added to a
regimen that already includes a long-acting bronchodilator (choice C). 
> Smoking cessation continues to be the most important therapeutic intervention for
COPD (choice D). 
> Long-term O2 (choice E) therapy prolongs life in COPD patients whose Pao2 is
chronically < 55 mm Hg. Continual 24-h use is more effective than a 12-h nocturnal
regimen. O2 therapy brings Hct toward normal levels; moderately improves
neuropsychologic factors, possibly by facilitating sleep; and ameliorates pulmonary
hemodynamic abnormalities. O2 therapy also increases exercise tolerance in many
patients. Long-term oxygen therapy improves survival by 2-fold or more in
hypoxemic patients with COPD

Clubbing (choice B) is an enlargement of the tips of the fingers or toes and a change
in the angle where the nails emerge. Clubbing seems to occur with some lung
disorders (lung cancer, lung abscess, bronchiectasis), but not with others (pneumonia,
asthma, chronic obstructive pulmonary disease). Clubbing also occurs with some
congenital heart disorders and liver disorders, or in some cases, it may be inherited
and not indicate any disorder. Clubbing itself does not need treatment.

Question #29
QID: 1935
Topic: COPD
Subject: Medicine

An 83-year-old male has a long history of COPD. His resting oxygen saturation is
86% on room air. Treatment includes oral bronchodilators, inhaled corticosteroids,
inhaled beta-agonists, inhaled anticholinergics, and home oxygen.

Which one of his treatments has been shown to prolong survival in cases such as this?

a)  Oral bronchodilators
b)  Inhaled corticosteroids
c)  Inhaled beta-agonists
d)  Inhaled anticholinergics
e)  Home oxygen

The correct answer is e)

Explanation:
Treatment of hypoxemia is critical in the management of COPD and trials have
shown a reduction in mortality with the use of oxygen for 15 or more hours daily.

> Inhaled beta-adrenergic agonists and anticholinergic agents, either alone or in


combination, provide symptomatic relief but do not prolong survival.
> Theophylline can be used for symptoms inadequately relieved by bronchodilators.
> Inhaled corticosteroids do not appear to alter the rate of decline in lung function in
COPD. However, some evidence shows that these agents alleviate symptoms and
reduce disease exacerbation. Pulmonary rehabilitation improves quality of life and
reduces hospitalizations.

Question #30
QID: 2754
Topic: COPD
Subject: Medicine

A 73-year-old white male has severe COPD manifested by repeating hospital


admissions, 30-step dyspnea on exertion, asthenia, and a resting pO2 of 58 mm Hg.

Of the following, which intervention will most likely positively affect his survival?  

a)  Beta2-Agonists
b)  Inhaled corticosteroids
c)  Oxygen supplementation
d)  Pulsed antibiotic therapy

The correct answer is c)

Explanation:
For patients afflicted with severe COPD, as evidenced by an FEV1 of 1.0 or less and
PaO2 of 60 mm Hg or less, only supplemental oxygen has been shown to positively
affect survival, reduce dyspnea scores, and reduce pulmonary artery pressure. Beta-
Agonists and inhaled corticosteroids, especially when used in combination, are
associated with a lower rate of exacerbations, but have no direct effect on mortality.
Smoking cessation, while clearly of benefit to all, is the only therapy to slow the
decline in lung function but does not affect survival. Finally, there is no compelling
study showing any impact on survival with pulsed antibiotic therapy.

Question #31
QID: 2931
Topic: COPD
Subject: Medicine

Which one of the following is most effective in slowing lung function decline in
COPD?  

a)  Theophylline 
b)  Inhaled β-agonists 
c)  Inhaled corticosteroids 
d)  Antibiotics 
e)  Smoking cessation

The correct answer is e)

Explanation:
In patients with COPD, smoking cessation has been shown to slow the decline of lung
function and even improve lung function. Theophylline, inhaled β-agonists, inhaled
corticosteroids, and antibiotics will not provide this benefit.

Question #32
QID: 3267
Topic: COPD
Subject: Medicine

Which one of the following is most effective in limiting the complications of


COPD?  

a)  Pneumococcal vaccination
b)  Smoking cessation
c)  Oral corticosteroids
d)  Albuterol
e)  Theophylline

The correct answer is b)

Explanation:
Smoking cessation is the only intervention that has been shown to slow the
progression of COPD and limit complications. Albuterol and theophylline will
improve acute problems, but will not slow disease advancement. Corticosteroids are
not indicated for chronic management. Although pneumococcal vaccine is
administered frequently, there is very little evidence to support a direct benefit in
preventing complications of COPD.

Question #33
QID: 5387
Topic: COPD
Subject: Medicine

Mr. Smith, a 55 year old male, has been your patient for 10 years. He is a heavy
smoker and has been diagnosed with COPD 6 years ago. He has been admitted to the
hospital multiple times for COPD exacerbations. Which of the following interventions
would you like to discuss with Mr. Smith because of the evidence based fact that it
prolongs survival in COPD patients?
a)  Albuterol inhaler
b)  Antibiotic prophylaxis
c)  Home oxygen
d)  Ipratropium inhaler
e)  Steroids

The correct answer is c)

Explanation:
After quitting smoking, home oxygen therapy is the only modality known to prolong
survival in COPD. Home oxygen therapy is recommended when PaO2 < 55 mm Hg
or oxygen saturation < 88%. Hypoxia is the major cause of mortality in COPD
patients.

Survival issues are becoming hot topics for the licensing exams.

Bronchodilators control the symptoms of COPD patients but have not been shown to
improve survival.

Antibiotics and steroids are recommended for acute exacerbations. They have no role
for chronic COPD management.

Question #34
QID: 5871
Topic: COPD
Subject: Medicine

A 72 year old woman with a history of COPD and arthritis presents to the E.R. with
difficulty breathing. Her caregiver tells you she is “feverish”, breathing harder than
normal and coughing up much more sputum which has turned thick and green. Your
management plan should consist of:
a)  ABC’s, Supplemental O2 and bronchodilation
b)  ABC’s, BiPAP
c)  ABC’s, O2, bronchodilators, systemic steroids, antibiotics
d)  ABC’s, Systemic steroids and antibiotics
e)  Immediate intubation and admission to the ICU

The correct answer is c)

Explanation:
COPD Exacerbation

A COPD exacerbation can be triggered by a range of factors such as viral/bacterial


infections, environmental irritants, CHF & PE. Exacerbations are indicated by an
increase in coughing and sputum production/volume from baseline, sputum
purulence; wheezing, chest tightness and fevers may also present.

Investigations should include: CBC, Serum chemistries, ABG (if severe), ECG, CXR,
Blood/Sputum cultures

Management should consist of:


1. An assessment of ABC’s;
2. Supplemental O2;
3. Bronchodilators by nebulizer (e.g. salbutamol, ipatropium);
4. Systemic corticosteroids (e.g. IV solumedrol);
5. Antibiotics (if indicated: i.e., purulent sputum, elevated WBC’s, fevers...etc)
e.g. doxycycline, trimethoprim-sulfamethoxazole and amoxicillin
clavulanate to cover the usual suspects (Streptococcus pneumoniae,
Haemophilus influenzae and Moraxella catarrhalis).

BiPaP (b) is reserved for severe exacerbations.


Option (e) is indicated in patients with deteriorating arterial blood gas (ABG) values,
altered mental status (AMS) and progressive respiratory fatigue.

PEARL: Management of COPD exacerbation should include: ABC’s, O2,


bronchodilators, systemic steroids, and antibiotics.

Question #35
QID: 6097
Topic: COPD
Subject: Medicine

Which one of the following is recommended in the treatment of all four stages of
COPD, from mild through very severe?
a)  Scheduled oral mucolytics such as N-acetylcysteine (Mucomyst)
b)  Scheduled inhaled corticosteroids such as fluticasone (Flovent HFA)
c)  Scheduled long-acting inhaled bronchodilators such as salmeterol (Serevent)
d)  Scheduled long-acting anticholinergics such as tiotropium (Spiriva)
e)  Short-acting inhaled beta-agonists such as albuterol (Ventolin HFA), as needed for
dyspnea

The correct answer is e)

Explanation:
Short-acting bronchodilators such as albuterol and ipratropium are recommended on
an as-needed basis for treatment of breathlessness in stage I (mild) COPD. They are
also recommended for as-needed use in stage II (moderate), stage III (severe), and
stage IV (very severe) COPD.

> Mucolytics can be considered for stages III and IV. 


> Inhaled corticosteroids are recommended for stages III and IV. 
> Long-acting bronchodilators such as salmeterol or tiotropium are recommended for
stages II, III, and IV.

Question #36
QID: 8786
Topic: COPD
Subject: Medicine

A 66-year-old male smoker is being evaluated for a persistent cough and difficulty
breathing. Spirometry confirms a fixed obstructive pathology with an FEV of about
50% of predicted for his size and age. His oxygen saturation is 89%-90% on room air.

Which one of the following would be most effective to prevent worsening of this
patient’s condition?

a)  A combined inhaled corticosteroid and long-acting Beta-agonist


b)  A long-acting anticholinergic agent
c)  Long-term oral corticosteroids
d)  Oxygen therapy
e)  Smoking cessation

The correct answer is e)

Explanation:
This patient has moderate to severe COPD. Smoking cessation is the single most
important therapeutic intervention in patients with this condition and should be a
priority of care. No existing medications have been shown to modify the long-term
decline in lung function that is typical of COPD, but smoking cessation does prevent
this decline. Long-term use of oxygen in COPD patients who also have chronic,
severe hypoxia (<88% saturation) can improve quality of life and prolong survival;
however, oxygen cannot prevent further decline in lung function. Long-term use of
oral corticosteroids is discouraged because of an unfavorable risk-to-benefit ratio.

Question #37
QID: 439
Topic: Extrinsic Allergic Alveolitis
Subject: Medicine

Extrinsic allergic alveolitis may be acquired by breathing air contaminated by which


one of the following substances?  

a)  Fine silicaceous dust


b)  Degreasing solvent
c)  Fungal spores
d)  Urea formaldehyde foam insulation
e)  Chlorine gas
The correct answer is c)

Explanation:
Extrinsic allergic alveolitis (hypersensitivity pneumonitis) is a type of inflammation in
and around the tiny air sacs (alveoli) and smallest airways (bronchioles) of the lung
caused by an allergic reaction to inhaled organic dusts or, less commonly, chemicals.

It can be caused by sensitization to many organic dusts mainly fungal spores. Many
types of dust can cause allergic reactions in the lungs. Organic dusts that contain
microorganisms or proteins and chemicals, such as isocyanates, may cause
hypersensitivity pneumonitis. Farmer's lung, which results from repeated inhalation of
heat-loving (thermophilic) bacteria in moldy hay, is a well-known example of
hypersensitivity pneumonitis.

Question #38
QID: 495
Topic: Asthma
Subject: Medicine

Which of the following is not given in an acute attack of asthma?  

a)  Ipratropium bromide
b)  Sodium cromolyn
c)  Oxygen
d)  Albuterol
e)  Prednisone

The correct answer is b)

Explanation:
Asthma is a disease of diffuse airway inflammation caused by a variety of triggering
stimuli resulting in partially or completely reversible bronchoconstriction. Symptoms
and signs include dyspnea, chest tightness, and wheezing.

The diagnosis is based on history, physical examination, and pulmonary function


tests. Treatment involves controlling triggering factors and drug therapy, most
commonly with inhaled β-agonists and inhaled corticosteroids. Anticholinergic
medications, such as ipratropium bromide provide addition benefit when used in
combination with beta-agonists in those with moderate or severe symptoms.

> Cromolyn is used more in the prophylaxis of asthma.

Question #39
QID: 529
Topic: Asthma
Subject: Medicine

An asthmatic patient presents to the ED with dyspnea. He states he used his inhaled
salbutamol with no improvement. Physical exam shows dyspnea with rhonchi in the
upper chest.
What is the best next step in management?
a)  IV fluids and aminophylline
b)  Oral Corticosteroids
c)  Endotracheal intubation
d)  IV methylprednisolone
e)  Add Ipratropium

The correct answer is b)

Explanation:
The best strategy for management of acute exacerbations of asthma is early
recognition and intervention, before attacks become severe and potentially life
threatening.

The basic principles of care are the following:

 Assess the severity of the attack


 Use inhaled short-acting beta agonists early and frequently and consider
concomitant use of ipratropium for severe exacerbations
 Start systemic glucocorticoids if there is not an immediate and marked
response to the inhaled short-acting beta agonists
 Make frequent (every one to two hours) objective assessments of the response
to therapy until definite, sustained improvement is documented
 Admit patients who do not respond well after four to six hours to a setting of
high surveillance and care
 Educate patients about the principles of self-management for early recognition
and treatment of a recurrent attack and develop an "asthma action plan" for
recurrent symptoms.

Since this patient is not responding to a short acting beta agonist, systemic
corticosteroids should be employed (choice B). Ipratropium (choice E) has had
variable benefit in controlled trials, demonstrating most consistent efficacy in children
and smokers with comorbid COPD. The addition of ipratropium has not been shown
to provide further benefit once the patient is hospitalized.

Note: Although use of systemic corticosteroids is recommended early in the course of


acute exacerbations in patients with an incomplete response to beta agonists, oral
administration is equivalent in efficacy to intravenous administration (choice D).
Question #40
QID: 829
Topic: Asthma
Subject: Medicine

An anxious 40-year-old patient, under long-term therapy for bronchial asthma, is


admitted to the Emergency Department because of status asthmaticus. Therapy should
include which one of the following?  

a)  Inhaled beta-2 agonists, hydration, ipratropium bromide


b)  Parenteral aminophylline, sodium cromoglycate
c)  Hydration, inhaled beta-2 agonists, parenteral corticosteroids
d)  Parenteral corticosteroids, sedative
e)  Parenteral aminophylline, parenteral corticosteroids, sedative

The correct answer is c)

Explanation:
The most severe form of asthma is called status asthmaticus. In this condition, the
lungs are no longer able to provide the body with adequate oxygen or adequately
remove carbon dioxide. Without oxygen, many organs begin to malfunction. The
buildup of carbon dioxide leads to acidosis, an acidic state of the blood that affects the
function of almost every organ. Blood pressure may fall to low levels. The airways
are so narrowed that it is difficult to move air in and out of the lungs.

Patients who have status asthmaticus receive a beta-adrenergic agonist and


corticosteroids. Status asthmaticus may also require intubation and ventilator support.

Question #41
QID: 1159
Topic: Asthma
Subject: Medicine

A 30 year old patient with asthma complains of daily wheezing and occasional
waking at night with cough and chest tightness for three weeks. His usual medication
is salbutamol two puffs tid-qid.

The next step in management is:  

a)  Add long-term theophylline


b)  Increase salbutamol to two puffs q4h
c)  Add ipratropium bromide two puffs qid
d)  Add beclomethasone two puffs qid
e)  Discontinue salbutamol and begin prednisone 50 mg and taper over 2 weeks

The correct answer is d)


Explanation:
Stage one of asthma is when a patient has symptoms 2 or less times per week. For
these patients the use of a short acting beta agonist (eg albuterol, salbutamol) on an as
needed basis is recommended.

Stage two of asthma is when a patient has symptoms greater than twice a week. At
this point you must add as inhaled steroid (eg fluticasone, beclomethasone) to the
patients treatment. Some physicians will also add a nighttime dose of singulair.

Question #42
QID: 2016
Topic: Asthma
Subject: Medicine

Treatment of mild persistent asthma with a low-dose inhaled corticosteroid alone


would be the preferred treatment for a patient whose asthma symptoms occur:  

a)  Once a day
b)  3 times a week and less than once a day
c)  Once a week
d)  Several times a day
e)  2 times a week, but only with exercise

The correct answer is b)

Explanation:
Low-dose inhaled corticosteroids are preferred for patients with mild persistent
asthma, defined as the occurrence of symptoms more than twice a week but do not
occur every day; nighttime symptoms occur 3 to 4 times a month. Patients with severe
persistent asthma, defined as continual symptoms, should be treated with high-dose
inhaled corticosteroids and long-acting inhaled Beta2-agonists. Moderate asthma,
defined as daily symptoms, should be treated with low- to medium-dose inhaled
corticosteroids and long-acting inhaled Beta2-agonists. Mild intermittent asthma, with
symptoms 1 or 2 days per week, does not require daily medication.

Question #43
QID: 2905
Topic: Asthma
Subject: Medicine

A 45-year-old female with asthma uses her albuterol (Proventil, Ventolin) inhaler
“once or twice a week” to relieve her wheezing. What daily medication should now
be added?  
a)  No daily medication  
b)  Low-dose inhaled corticosteroids 
c)  Oral β2-agonists 
d)  Long-acting inhaled β2-agonists 
e)  Low-dose inhaled corticosteroids plus long-acting inhaled β2-agonists

The correct answer is a)

Explanation:
The patient has mild intermittent asthma and requires no daily medication. Current
guidelines recommend the following treatments based on asthma classifications:

Mild intermittent asthma (symptoms 2 days per week or less, nighttime symptoms 2
nights or less per month) - no daily medication needed.

Mild persistent asthma (symptoms more than 2 days per week but less than one time
per day. nighttime symptoms more than 2 nights per month) - daily low-dose inhaled
corticosteroids.

Moderate persistent asthma (symptoms every day, nighttime more than once weekly)
- daily low- to medium-dose inhaled corticosteroids and long-acting β2-agonists.

Severe persistent asthma (symptoms continual during the day, frequently at night) -
daily high-dose inhaled corticosteroids and long-acting β2-agonists.

Question #44
QID: 2954
Topic: Asthma
Subject: Medicine

A 34-year-old black female has had frequent asthma exacerbations for the past several
months. She uses her albuterol (Proventil, Ventolin) inhaler 4 times per week with a
good response.

Which one of the following is preferred for the prevention of future asthma
exacerbation?  

a)  Long-acting β2-agonists 
b)  Leukotriene modifiers 
c)  Inhaled corticosteroids  
d)  Theophylline 
e)  No additional therapy

The correct answer is c)


Explanation:
This patient has mild persistent asthma and a controller medicine is indicated. The
preferred treatment for patients with mild persistent asthma is low-dose inhaled
corticosteroids. Alternate second-line treatments include cromolyn, leukotriene
modifiers, nedocromil, and sustained-release theophylline. Long-acting β2-agonists
should not be used alone for the treatment of persistent asthma. They can be used in
combination with inhaled corticosteroids for initial treatment of moderate persistent
asthma, but are not recommended for initial treatment of mild persistent asthma.
Short-acting bronchodilators, such as albuterol, are effective for treatment of acute
asthma symptoms, but should not be used as preventive treatment of patients with
persistent asthma.

Question #45
QID: 3214
Topic: Asthma
Subject: Medicine

Which one of the following is most consistent with a diagnosis of asthma?  

a)  Reduced FEV1 and reduced FEV1 /FVC ratio


b)  Reduced FEV1 and normal FEV1 /FVC ratio
c)  Reduced FEV1 and increased FEV1 /FVC ratio
d)  Reduced FVC and normal FEV1 /FVC ratio
e)  Reduced FVC and increased FEV1 /FVC ratio

The correct answer is a)

Explanation:
Asthma is typically associated with an obstructive impairment that is reversible with
short-acting bronchodilators. A reduced FEV1 and decreased FEV1 /FVC ratio
indicates airflow obstruction.
A reduced FVC with a normal or increased FEV1 /FVC ratio is consistent with a
restrictive pattern of lung function.

Question #46
QID: 5320
Topic: Asthma
Subject: Medicine

A 35-year-old male presents with cough, breathlessness, wheeze and tightness in his
chest. He states that all this has started after 15mins of his afternoon jogging. What is
the most likely diagnosis of his condition?
a)  Emphysema
b)  Allergies
c)  Asthma
d)  Bronchitis
e)  Unstable Angina

The correct answer is c)

Explanation:
Answer: C - Asthma is characterized by cough, breathlessness, wheezing and
tightness of chest. Since the patient suffered from the symptoms after exercise, it is
known as exercise induced asthma. Asthma is a chronic inflammatory disorder of the
airways due to inflammatory responses or airway obstruction or hyperresponsiveness
or air way remodeling. Factors which precipitate the asthma are allergens, chemical
irritants, flour, wood, textile chemicals, viral infections, exercise, emotions and few
drugs. Allergens like pollen are the most common allergen responsible for the cause
of asthma. Chemical irritants like industrial pollutants are also responsible for
precipitating the asthma. Depression, anxiety and excess laughing also trigger
asthma. 

A. Emphysema is one of the chronic obstructive pulmonary diseases marked by lysis


of the alveoli resulting in formation of sac like structures.
B. Allergies present differently (teary eyes, runny nose...usually not with sudden
onset)
D. Bronchitis is unlikely due to presentation. (not exercise induced, usually
accompanied by fever, sore throat etc)
E. Unstable Angina usually occurs at rest or with minimal exercise and does not
present with cough or wheezing.

Question #47
QID: 5321
Topic: Asthma
Subject: Medicine

A 35 year old male presents with cough, breathlessness, wheeze and tightness in his
chest. He states that all this has started after 15mins of his afternoon jogging. Which
of the following should be obtained as the primary test to establish this patient's
diagnosis?
a)  X-rays
b)  Peak flow rate
c)  Allergy test
d)  Spirometry
e)  Pulse oximetry

The correct answer is d)


Explanation:
Answer: D - Spirometry assessments should be obtained as the primary test to
establish the asthma diagnosis. Spirometry should be performed prior to initiating
treatment in order to establish the presence and determine the severity of baseline
airway obstruction.
Spirometry gathers many measurements; the primary ones are:
-the forced expiratory volume in the first second (FEV1);
-the forced vital capacity (FVC);
-the ratio of the FEV1 divided by the FVC (the FEV1/FVC or more simply the
FEV1%);
-the peak expiratory flow (PEF); and
-the forced expiratory time (FET100%).

A. In most patients with asthma, chest radiography findings are normal or may
indicate hyperinflation.
B. Peak-flow monitoring is designed for ongoing monitoring of patients with asthma
because the test is simple to perform and the results are a quantitative and
reproducible measure of airflow obstruction. Peak-flow monitoring should not be
used as a substitute for spirometry to establish the initial diagnosis of asthma.
C. Allergy skin testing is a useful adjunct in individuals with atopy. Results help
guide indoor allergen mitigation or help diagnose allergic rhinitis symptoms.
E. Pulse oximetry is a non-invasive way to measure oxygenation of blood or how well
oxygen is being exchanged between the lungs and the blood. It is not diagnostic.

Question #48
QID: 5637
Topic: Asthma
Subject: Medicine

A 24-year-old female with a past history of asthma presents to the emergency


department with an asthma exacerbation. Treatment with an inhaled bronchodilator
and ipratropium (Atrovent) does not lead to significant improvement, and she is
admitted to the hospital for ongoing management.
On examination she is afebrile, her respiratory rate is 24/min, her pulse rate is 92
beats/min, and oxygen saturation is 92% on room air. She has diffuse bilateral
inspiratory and expiratory wheezes with mild intercostal retractions. Which one of the
following should be considered in the acute management of this patient?
a)  Chest physical therapy
b)  Inhaled fluticasone/salmeterol (Advair)
c)  Oral azithromycin (Zithromax)
d)  Systemic corticosteroids
e)  Oral theophylline

The correct answer is d)

Explanation:
Hospital management of acute exacerbations of asthma should include inhaled short-
acting bronchodilators in all patients. Systemic corticosteroids are recommended for
all patients admitted to the hospital. The efficacy of oral prednisone has been shown
to be equivalent to that of intravenous methylprednisolone. Oxygen should also be
considered in most patients. Antibiotics are not recommended in the treatment of
asthma exacerbations unless there is a comorbid infection. Inhaled ipratropium
bromide is recommended for treatment in the emergency department, but not in the
hospital. Chest physical therapy and methylxanthines are not recommended in the
treatment of acute asthma exacerbations.

Question #49
QID: 564
Topic: Pulmonary Edema
Subject: Medicine

A patient presents with pulmonary edema and has a blood pressure of 180/95 mm Hg.
What is the most appropriate initial treatment?

a)  Furosemide
b)  Metoprolol
c)  Morphine
d)  Enalapril

The correct answer is a)

Explanation:
Sometimes increased pressure in the blood vessels in the lungs forces fluid into the air
sacs, preventing them from absorbing oxygen, a condition called pulmonary
edema. Intravenous diuretics are first-line medications to help the kidneys remove
excess fluid from the body.

The medical treatment of pulmonary edema is as follows:

- Preload reducers. Diuretics, such as furosemide (Lasix) are used to treat pulmonary
edema. Loop diuretics are presumed to decrease preload through 2 mechanisms:
diuresis and direct vasoactivity (venodilation).
- Morphine, a narcotic, is a mainstay in treating cardiac pulmonary edema, may be
used as anxiolytic, with a resulting decrease in catecholamine production and a
decrease in systemic vascular resistance.
- Afterload reducers. Some examples of afterload reducer medications include
nitroprusside and enalapril. Nitroprusside results in simultaneous preload and
afterload reduction by causing direct smooth-muscle relaxation, with an increased
effect on afterload. The hemodynamic effects of ACE inhibitors include reduced
afterload, improved stroke volume and cardiac output, and a slight reduction in
preload. The last effects happen when renal perfusion improves after cardiac output
improves and diuresis occurs.
Question #50
QID: 2918
Topic: Pulmonary Edema
Subject: Medicine

Which one of the following is most characteristic of noncardiogenic pulmonary


edema?  

a)  Paroxysmal nocturnal dyspnea 


b)  Poor left ventricular function 
c)  Brain natriuretic peptide levels >500 pg/mL 
d)  An S3 gallop 
e)  A history of sepsis, trauma, or aspiration

The correct answer is e)

Explanation:
Increased membrane permeability in the lungs causes noncardiogenic pulmonary
edema, flooding the lungs with fluid from the vascular space. Consequently, it is
associated with damage to the lungs from trauma, aspiration, or sepsis.

Paroxysmal nocturnal dyspnea, an S3 gallop, and poor left ventricular function are
associated with cardiac-induced pulmonary edema. Elevated brain natriuretic peptide
levels mean that the cardiac walls are stretched and that the heart is likely the cause of
the pulmonary edema.

Question #51
QID: 5388
Topic: Pulmonary Edema
Subject: Medicine

A 70 year old man presents to the ED with a severe shortness of breath and
diaphoresis.

His past medical history is significant for Diabetes and two previous MIs. He is taking
metformin, pioglitazone, aspirin, losartan and rosuvastatin. His vitals are significant
for a pulse rate of 110/min and a RR of 30/min. Physical exam shows JVD, lower
limb pitting edema and bilateral basilar lung crackles.

Which of the following medications is the most appropriate at this time?


a)  Beta blockers
b)  Digoxin
c)  Lidocaine
d)  Mannitol
e)  Morphine

The correct answer is e)

Explanation:
The best initial therapy for Acute cardiogenic pulmonary edema includes Loop
diuretics, Morphine, Nitrates and Oxygen “LMNO”. Morphine promotes
venodilation thus decreases the preload; moreover, morphine alleviates the severe
anxiety of acute pulmonary edema patients.

Beta blockers are excellent drugs for congestive heart failure and decrease mortality;
however they should be avoided in the acute exacerbation because of its negative
inotropism.

Osmotic diuretics such as mannitol are contraindicated because they increase the
blood volume thus the afterload.

Digoxin would be appropriate for long term management.

In the absence of a ventricular tachycardia or other arrhythmia, lidocaine is not


recommended.

Question #52
QID: 679
Topic: Sarcoidosis
Subject: Medicine

A 30-year-old woman complains to of swollen lymph nodes in her neck and painful
nodules on her lower legs. A chest x-ray shows bilateral hilar and right paratracheal
lymphadenopathy. Which one of the following would be the most likely diagnosis if
erythema nodosum were added to the above presentation?  

a)  Lymphoma
b)  Metastatic thyroid carcinoma
c)  Teratoma
d)  Wegener's granulomatosis
e)  Sarcoidosis

The correct answer is e)

Explanation:
Sarcoidosis is characterized by noncaseating granulomas in one or more organs and
tissues; etiology is unknown. The lungs and lymphatic system are most often affected,
but sarcoidosis may affect any organ. Pulmonary symptoms range from none (limited
disease) to exertional dyspnea and, rarely, lung or other organ failure (advanced
disease).

Diagnosis usually is first suspected because of pulmonary involvement and is


confirmed by chest x-ray, biopsy, and exclusion of other causes of granulomatous
inflammation. Chest x-ray, or high-resolution chest CT will show hilar and
mediastinal lymphadenopathy. First-line treatment is corticosteroids. Prognosis is
excellent for limited disease but poor for more advanced disease.

Question #53
QID: 1884
Topic: Sarcoidosis
Subject: Medicine

You have just diagnosed pulmonary sarcoidosis in a 35 year old black female. Which
one of the following should be performed on the patient?

a)  Slit lamp examination of the eyes


b)  HLA serotype testing
c)  Helper-to-suppressor T-cell ratio
d)  Kveim skin test

The correct answer is a)

Explanation:
Patients with sarcoidosis should have a slit lamp examination to rule out uveitis,
pulmonary function testing, a serum calcium level, and electrocardiography. In
addition, serum angiotensin converting enzyme and anergy skin testing might be
considered. Certain HLA types have been associated with sarcoidosis in Japan, but
testing is not specific. T-cell ratios may explain the reason for anergy, but are not
specific or recommended. The Kveim skin test is not used clinically now because of
the danger of injecting sarcoid antigen into another patient.

Question #54
QID: 2966
Topic: Sarcoidosis
Subject: Medicine

A 25-year-old clinically healthy black female is involved in a minor auto accident.


Chest radiographs obtained after the wreck reveal bilateral hilar lymphadenopathy.
She has no history of environmental exposures and has no symptoms. A physical
examination is completely normal. Your initial workup includes a normal
comprehensive metabolic panel, CBC, and urinalysis; a negative tuberculin skin test;
a normal EKG; and normal pulmonary function tests. A transbronchial lung biopsy
specimen reveals a noncaseating epithelioid granuloma.

Which one of the following would be the most appropriate treatment at this time?  
a)  Long-term high-dose systemic corticosteroids 
b)  Pulsed doses of systemic corticosteroids 
c)  Inhaled corticosteroids 
d)  Oral methotrexate weekly 
e)  Observation only

The correct answer is e)

Explanation:
Sarcoidosis is a disease of unknown cause characterized by the presence of
noncaseating epithelioid granulomas; it involves many different organ systems. The
lungs are commonly involved, with bilateral hilar lymphadenopathy being commonly
present and pulmonary infiltrates and fibrosis somewhat typical. Other frequent areas
of involvement include the skin, central nervous system, eyes, liver, heart, salivary
glands, kidneys, muscles, and bones. When the disease is limited to asymptomatic
hilar adenopathy, it is termed stage I and no treatment has been shown to be
beneficial. The most appropriate management of stage I patients is routine follow-up.

Question #55
QID: 5417
Topic: Sarcoidosis
Subject: Medicine

A 25-year-old, African-Canadian female presents to your clinic with the complaints of


shortness of breath for the past 3 months.

She said that the shortness of breath was mild at the beginning but has worsened
progressively over the last 3 months. One month ago, she had painful reddish lesions
on both legs that resolved spontaneously. Her review of systems reveals intermittent
joint stiffness. She denies any weight loss, night sweats, recent infection, travel or
exposure to ill contacts. She does not smoke but drinks alcohol on social occasions.
Her only medication is a daily multi-vitamin formula.

Vital signs are within normal limits. Breath sounds are clear to auscultation
bilaterally. The heart exam is regular in rate and rhythm with normal S1 and S2 and
no murmurs, rubs or gallops.

You order a Chest X-ray which reveals symmetric bilateral hilar lymphadenopathy
and diffuse interstitial infiltrates. Labs, including CBC, ESR and BMP, are normal.
Which of the following is the most likely diagnosis?
a)  Atypical pneumonia
b)  Fibrosing alveolitis
c)  Lymphoma
d)  Sarcoidosis
e)  Tuberculosis

The correct answer is d)

Explanation:
Sarcoidosis is a non-caseating granulomatous multisystem disorder of unknown
etiology. Bilateral symmetric hilar lymphadenopathy in a young African person is a
classic presentation. It can also involve any system in the body; pulmonary interstitial
infiltrates, liver involvement, skin or eye disease (uveitis). The patient’s history of
painful reddish lesions on her legs is characteristic of erythema nodosum associated
with sarcoidosis.

Lymphoma is a big differential and should be ruled out with a transbronchial biopsy;
however, the African race, the symmetric pattern of hilar lymphadenopathy and the
absence of systemic symptoms make lymphoma less likely.

In the absence of risk factors, cough, weight loss, night sweats and fever, TB is
unlikely.

Fibrosing alveolitis (idiopathic pulmonary fibrosis - IPF) is less likely because of the
presence of extrapulmonary manifestations (joint stiffness) and the bilateral
lymphadenopathy. Moreover, IPF usually presents in older ages.

Atypical pneumonia would give fever, crackles on physical exam without the hilar
lymph nodes involvement.

Question #56
QID: 6146
Topic: Sarcoidosis
Subject: Medicine

A 23-year-old black male presents with a history of several weeks of dyspnea, cough
productive of bloody streaks, and malaise. His examination is normal except for
bilateral facial nerve palsy. A CBC and urinalysis are normal. A chest radiograph
reveals bilateral lymph node enlargement.

This presentation is most consistent with:


a)  Polyarteritis nodosa
b)  Goodpasture’s syndrome
c)  Sarcoidosis
d)  Pulmonary embolus

The correct answer is c)

Explanation:
Sarcoidosis, a disease of unknown etiology, affects young to middle-age adults
(predominantly 20-29 years old). In Canada it is more common in blacks. It is
asymptomatic in 30%-50% of patients, and is often diagnosed on a routine chest film.
About one-third of cases will present with fever, malaise, weight loss, cough, and
dyspnea. The pulmonary system is the main organ system affected, and findings may
include bilateral hilar lymphadenopathy and discrete, noncaseating epithelial
granulomas. Facial nerve palsy is seen in <5% of patients, and usually occurs late in
the process. Before Lyme disease was recognized, bilateral facial nerve palsy was
almost always due to sarcoidosis. Hemoptysis does not generally occur until late in
the course of sarcoidosis, and is usually related to Aspergillus infection or cavitation.
Renal involvement rarely results in significant proteinuria or hematuria.

> Polyarteritis nodosa may involve the lungs, but rarely. Although pneumonic
episodes may be associated with hemoptysis in a small percentage of patients, the
chest radiograph is more likely to reveal granulomatous lesions rather than patchy
infiltrates.
> Goodpasture’s syndrome is characterized by pulmonary hemorrhage,
glomerulonephritis, and antiglomerular basement membrane antibodies. Hemoptysis,
pulmonary alveolar infiltrates, dyspnea, and iron-deficiency anemia are frequent
presenting features. Within days or weeks, the pulmonary findings are generally
followed by hematuria, proteinuria, and the rapid loss of renal function.
> Pulmonary embolus is an acute event, and would present with dyspnea and possibly
hemoptysis, but not hilar lymphadenopathy.

Question #57
QID: 10832
Topic: Sarcoidosis
Subject: Medicine

A 32-year-old black female presents to your department for a post-job offer medical
examination. As part of the screening an X-ray is ordered and the image is shown
here:

The patient denies cough, shortness of breath, sore joints, fever, chills, or sweats.
On physical examination the patient has enlarged non-tender supraclavicular lymph
nodes but has no facial skin lesions, no hepatosplenomegaly, and no erythema
nodosum. A PPD test is performed and is negative. Culture of bronchoalveolar lavage
fluid is negative for growth and Gram’s stain only shows normal oral flora.
Pulmonary function tests and ECG are normal. What is the best next step in
management of this patient?
a)  No treatment, do periodic chest X-ray and pulmonary function tests
b)  Hospital admission and treatment with intravenous immunoglobulin
c)  Outpatient treatment with Isoniazid. Do sputum studies when patient is
symptomatic
d)  Treatment with oral prednisone and monitor supraclavicular lymph nodes' size
e)  Order complete blood cell count, do lymph node biopsy, and treat with
methotrexate

The correct answer is a)

Explanation:
This patient is asymptomatic but her diagnosis can still be made based on physical
examination and imaging findings. This patient’s most likely diagnosis is sarcoidosis.
It is a multisystem inflammatory disease of unknown etiology characterized by
noncaseating granulomas, predominantly in the lungs and intrathoracic lymph nodes.
Prevalence is 10 times greater in black than white people. When symptoms are
present the most common are fever, anorexia, dyspnea, cough, chest pain, and
arthralgias. Sarcoidosis has 4 stages:

 Stage 0 patients have normal chest X-ray,


 Stage 1 patients have lymphadenopathy only,
 Stage 2 lymphadenopathy and lung parenchymal disease,
 Stage 3 patients have parenchymal lung disease only, and
 Stage 4 pulmonary fibrosis.

Based on this patient’s chest X-ray with bilateral hilar lymphadenopathy and
pulmonary opacities, she most likely has stage 2 sarcoidosis. The hilar region is the
lung roots region and consists of major bronchi and the pulmonary veins and arteries.
Hilar lymph nodes are not visible on a normal X-ray but bilateral hilar
lymphadenopathy and right paratracheal lymphadenopathy are classic findings in
sarcoidosis and are known as the Garland’s triad. Usually the left hilum is higher than
the right.
Since this patient is asymptomatic and has normal pulmonary function tests she
should not be treated. The best management option is to monitor her chest X-ray
regularly as well as doing pulmonary function tests (choice A).

> Hospital admission and treatment with intravenous immunoglobulin (choice B) is


incorrect. There’s no reason to admit this patient at this point and while there have
been some reports of successful use of intravenous immunoglobulin to treat
sarcoidosis, it is not standard treatment for this disease.
> Outpatient treatment with Isoniazid and sputum studies when patient is
symptomatic (choice C) would be done in a patient with primary tuberculosis. This
patient’s PPD test is negative and her chest X-ray is suggestive of sarcoidosis.
> Treatment with oral prednisone and monitor supraclavicular lymph nodes size
(choice D) is incorrect. While corticosteroids are the mainstay therapy for
symptomatic sarcoidosis, this patient’s current condition does not warrant treatment.
> Order complete blood cell count, do lymph node biopsy, and treat with
methotrexate (choice E) is incorrect. Lymph node biopsy has been suggested as one of
the cheapest ways to confirm diagnosis and also help rule out any other possibilities
such as lymphoma. However, methotrexate is only used to treat sarcoidosis refractory
to prednisone and would not be appropriate at this point.

Sarcoidosis is characterized by bilateral hilar lymphadenopathy and paratracheal


lymphadenopathy on chest X-ray; this is known as Garland's triad. Asymptomatic
patients with normal pulmonary function tests should not be treated; their X-ray and
pulmonary function tests should be monitored.

Question #58
QID: 709
Topic: Sleep Apnea
Subject: Medicine

A 40-year-old man complains of chronic fatigue for the past year. His wife states he
snores loudly and at times she thinks he seems to stop breathing.
Which one of the following is the most likely diagnosis?  

a)  Narcolepsy syndrome
b)  Catalepsy syndrome
c)  Sleep apnea syndrome
d)  Kleine-Levin syndrome
e)  Hypersomnia disorder

The correct answer is c)

Explanation:
Obstructive sleep apnea consists of episodes of partial or complete closure of the
upper airway that occur during sleep and lead to breathing cessation (defined as a
period of apnea > 10 sec). Symptoms include restlessness, snoring, recurrent
awakening, morning headache, and excessive daytime sleepiness.

Diagnosis is based on sleep history, and polysomnography. Treatment is with nasal


continuous positive airway pressure, oral appliances, and, in refractory cases, surgery.
Prognosis is good with treatment. Most cases remain undiagnosed and untreated and
are often associated with hypertension, heart failure, and injury or death from motor
vehicle crashes and other accidents resulting from hypersomnolence.

Question #59
QID: 1960
Topic: Sleep Apnea
Subject: Medicine

Sleep apnea is known to be associated with:  

a)  Hypertension
b)  Respiratory muscle dysfunction
c)  Carpal tunnel syndrome
d)  Hypercalcemia
e)  Previous tonsillectomy

The correct answer is a)

Explanation:
Sleep apnea is associated with hypertension, and may in fact be a cause of
hypertension. Improvement of sleep apnea can be associated with improved blood
pressure.

Question #60
QID: 3194
Topic: Sleep Apnea
Subject: Medicine

For adults with obstructive sleep apnea, which one of the following is the most
effective treatment?  

a)  Modafinil (Provigil)
b)  Weight reduction to achieve a body mass index (BMI) <30 kg/m2
c)  Mandible-positioning oral appliances
d)  Continuous positive airway pressure (CPAP)
e)  Uvulopalatal surgery

The correct answer is d)


Explanation:
Continuous positive airway pressure (CPAP) is the most effective treatment for
obstructive sleep apnea in adults. Weight reduction is certainly beneficial in obese
patients, but may not significantly decrease apneic episodes. Results with devices that
move the tongue or mandible forward are variable and inconsistent. While modafinil
has an indication for treating sleep apnea, its efficacy is inconsistent. Uvulopalatal
surgery often reduces snoring but may not reduce the frequency of apneic episodes
during sleep.

Question #61
QID: 6096
Topic: Sleep Apnea
Subject: Medicine

The most common presenting symptom of obstructive sleep apnea is:


a)  Excessive daytime sleepiness
b)  Snoring
c)  Morning headache
d)  Gastroesophageal reflux
e)  Enuresis

The correct answer is a)

Explanation:
The most common presenting symptom of obstructive sleep apnea is excessive
daytime sleepiness (SOR A). Other symptoms include snoring, unrefreshing or
restless sleep, witnessed apneas and nocturnal choking, morning headache, nocturia or
enuresis, gastroesophageal reflux, and reduced libido.

Question #62
QID: 6143
Topic: Sleep Apnea
Subject: Medicine

A 55-year-old obese male with hypertension and daytime somnolence is found to


have severe obstructive sleep apnea, with an apnea-hypopnea index of 32 on an
overnight polysomnogram.

Which one of the following is considered to be first-line therapy for this patient's
condition?
a)  Continuous positive airway pressure (CPAP)
b)  An oral dental appliance
c)  Uvulopalatopharyngoplasty
d)  Sleep positioning therapy
e)  Tracheostomy

The correct answer is a)

Explanation:
Patients with severe sleep apnea (apnea-hypopnea index >29) and concomitant
cardiovascular disease benefit the most from treatment for obstructive sleep apnea.
Because it is relatively easy to implement and has proven efficacy, continuous
positive airway pressure (CPAP) is considered first-line therapy for severe apnea.

Question #63
QID: 8794
Topic: Sleep Apnea
Subject: Medicine

Patients with obstructive sleep apnea have an increased risk for:

a)  Chronic renal failure


b)  Hypertension
c)  Hypokalemia
d)  Hypothyroidism
e)  Sepsis

The correct answer is b)

Explanation:
Obstructive sleep apnea-hypopnea syndrome is defined as the presence of at least five
obstructive events per hour with associated daytime sleepiness. It is present in 2%-4%
of the population. The prevalence in men is almost three times that seen in
premenopausal women and twice that of postmenopausal women. Other factors
associated with an increased prevalence are obesity, older age, and systemic
hypertension.

Question #64
QID: 11364
Topic: Sleep Apnea
Subject: Medicine

The major problem with the use of continuous positive airway pressure (CPAP) for
obstructive sleep apnea is:

a)   Sneezing
b)   Dry mucous membranes
c)   Continued snoring
d)   Rhinorrhea
e)   Noncompliance

The correct answer is e)

Explanation:
Sneezing, rhinorrhea, and dry mucous membranes are common problems associated
with the use of continuous positive airway pressure (CPAP) for obstructive sleep
apnea. However, the major problem with CPAP is lack of compliance; patients
complain of discomfort, claustrophobia, panic attacks, intrusiveness, difficulty
keeping the appliance in place, and noise from the machine.

Question #65
QID: 729
Topic: Amyloidosis
Subject: Medicine

After a long history of multiple myeloma, a 67-year-old male is noted to have


abundant acellular eosinophilic deposits around the pulmonary microvasculature at
autopsy. A congo red special stain demonstrates apple green birefringence.

Which one of the following conditions is the most likely explanation?  

a)  Sarcoidosis
b)  Histoplasmosis
c)  Amyloidosis
d)  Mesothelioma
e)  Silicosis

The correct answer is c)

Explanation:
Amyloidosis is any of a group of disparate conditions characterized by extracellular
deposition of various insoluble proteins. These proteins may accumulate locally,
causing relatively few symptoms, or widely, involving multiple organs and producing
severe multiorgan failure. Amyloidosis can be primary or be secondary to various
infectious, inflammatory, or malignant conditions. Rarely, it results from any of
several inherited metabolic defects. Diagnosis is by biopsy of affected tissue.

Amyloid deposits may be formed from at least 18 different proteins, including


immunoglobulin fragments. Amyloid deposits are metabolically inert but interfere
physically with organ structure and function. All stain positive with Congo red dye,
stain pink with hematoxylin and eosin, and have apple-green birefringence under
polarized light after Congo red staining. Amyloid deposits have a fibrillar, usually
rigid, and nonbranching ultrastructure.

Question #66
QID: 758
Topic: A-a Gradient
Subject: Medicine

An increase in the alveolar-arterial PO 2 difference may occur in:  

a)  Asthma
b)  Acute hypoventilation
c)  Anemia
d)  Mild exercise
e)  High altitude

The correct answer is a)

Explanation:
The difference between the calculated PaO and the measured PaO is called the
2 2

Alveolar-arterial PO difference or 'A-a Gradient' for short. The A-a gradient answers
2

the important question: Are the lungs transferring oxygen properly from the
atmosphere to the pulmonary circulation? If the A-a gradient is elevated, the answer is
no. If the A-a gradient is normal the answer is yes.

Causes of increased (A-a) difference:


Alveolar collapse (atelectasis), intaalveolar filling (pneumonia, pulmonary edema),
intracardiac shunt, vascular shunt within lungs, asthma, COPD, ILD, alveolar disease,
pulmonary vascular disease.

Hypoventilation alone can't produce increased (A-a) difference


Hemoglobin level has an ignorable effect on (A-a) difference
In high altitude the inspired O is decresed so the (A-a) difference has no increase
2

Question #67
QID: 792
Topic: Silicosis
Subject: Medicine

A 65-year-old male, who has been recently retired from a cement manufacturing
company in Manitoba, complains of shortness of breath, fatigue, and a chronic
progressive cough that is resistant to antibiotic treatment. An x-ray shows multiple
small coin-like lesions in the upper lobe of the right lung. The TB skin test was
negative twice.

Which one of the following is the most likely cause?  


a)  Legionellosis
b)  Tuberculosis
c)  Silicosis
d)  Dimorphic mycosis
e)  Atypical mycobacteria

The correct answer is c)

Explanation:
Silicosis is caused by inhalation of crystalline-free silica dust and is characterized by
nodular pulmonary fibrosis. Chronic silicosis initially causes no symptoms or only
mild dyspnea but over years can advance to involve most of the lung and cause
dyspnea, hypoxemia, pulmonary hypertension, and respiratory impairment. Diagnosis
is based on history and chest x-ray. No effective treatment exists except supportive
care and, for severe cases, lung transplantation.

Question #68
QID: 815
Topic: Acid Base Disorders
Subject: Medicine

A 74-year-old female is brought to the emergency room. She is unresponsive and


restless. Her breathing is laboured. Her family indicates that she has a history of
confusion and dementia. They also indicate that she ingested "anti-freeze" from the
garage. Blood gas results would most likely show:

a)  Mixed acid-base disturbance


b)  Respiratory acidosis
c)  Respiratory alkalosis
d)  Metabolic alkalosis
e)   Metabolic acidosis

The correct answer is e)

Explanation:
Ingestion of ethylene glycol (commonly used in antifreeze) leads to a metabolic
acidosis and severe damage to the central nervous system, heart, lungs, and kidneys.
The increased anion gap and osmolar gap are attributable to ethylene glycol and its
metabolites, oxalic acid, glycolic acid, and other organic acids. Lactic acid production
increases secondary to inhibition of the tricarboxylic acid cycle and altered
intracellular redox state. Diagnosis is facilitated by recognizing oxalate crystals in the
urine, the presence of an osmolar gap in serum, and a high-AG acidosis.
Question #69
QID: 842
Topic: ARDS
Subject: Medicine

A 35-year-old man with a recent history of severe multiple trauma is noted to have
dyspnea and is diagnosed as having adult respiratory distress syndrome (ARDS).

All of the following statements about the management of ARDS are correct, except:  

a)  Management may include artificial ventilation with endotracheal intubation and


oxygen therapy
b)  A sudden deterioration in a ventilated patient should raise the suspicion of
pneumothorax
c)  Early management may include oxygen by mask and careful monitoring of
respiratory rate and arterial blood gases
d)  Furosemide is routinely used to keep the lungs as dry as possible
e)  Frequent arterial blood gas determinations will be required to assess the adequacy
of therapy in any ventilated patient

The correct answer is d)

Explanation:
People with ARDS are treated in an intensive care unit. Successful treatment depends
on correcting the underlying cause; oxygen therapy, which is vital to correcting low
oxygen levels, is combined with treatment of the underlying cause.

If oxygen delivered by a face mask or nasal prongs does not correct the low blood
oxygen levels, or if excessively high doses of inhaled oxygen are required, a
ventilator must be used; this treatment is called mechanical ventilation. A ventilator
delivers oxygen-rich air under pressure using a tube inserted through the mouth into
the trachea. The pressure supplied by the ventilator during and after a breath opens
collapsed (atelectatic) regions of the lung and allows oxygen to move through the
walls of the injured lungs into the blood.

The pressure and volume of air that the ventilator delivers to the lungs with each
breath must be adjusted to help keep the small airways and alveoli open while
avoiding rupturing the fragile air sacs, which can lead to air accumulating around the
lung and collapsing it (called pneumothorax). Monitoring and adjusting the pressure
also ensures that the lungs do not receive an excessive concentration of oxygen, which
can damage the lungs and worsen ARDS.

In some cases, diuretic drugs may be needed to help remove fluid from the lungs.
Antibiotics are usually needed for people who develop bacterial pneumonia. Some
people may benefit from the use of intravenous corticosteroids in the later stages of
ARDS.
Question #70
QID: 3197
Topic: Bronchitis
Subject: Medicine

Which one of the following is true regarding acute bronchitis?

a)  The associated cough typically lasts more than 2 weeks


b)  Fever, tachypnea, and rales are typical of acute bronchitis in young patients
c)  ß-Agonists are indicated for treatment
d)  Antimicrobial agents are indicated for treatment
e)  Purulent sputum implies a bacterial etiology

The correct answer is a)

Explanation:
The cough of acute bronchitis typically lasts 2 weeks and frequently up to 20 days.
Fever, tachypnea, and rales in young patients are typical of pneumonia, not bronchitis.
Both antimicrobials and ß-agonists are of questionable benefit in patients with acute
bronchitis. Viruses are the most common cause of acute bronchitis, and purulent
sputum reflects desquamation of the airway, with the expectorated matter being
mostly epithelial cells.

Question #71
QID: 919
Topic: Pleural Effusion
Subject: Medicine

A 70-year-old man with a history of coronary heart disease and hypertension, presents
with a 3 week history of increasing shortness of breath. A chest x-ray shows a
moderate sized right pleural effusion. Thoracentesis shows the protein in the
pleura/serum ratio to be < 0.5, consistent with a transudate.

These results are compatible with a pleural effusion caused by which one of the
following?  

a)  Nephrotic syndrome
b)  Congestive heart failure
c)  Adenocarcinoma of the lung
d)  Hepatic vein thrombosis
e)  Pneumonia

The correct answer is b)

Explanation:
Transudative effusions are caused by some combination of increased hydrostatic
pressure and decreased oncotic pressure in the pulmonary or systemic circulation.
Heart failure is the most common cause, followed by cirrhosis with ascites and
hypoalbuminemia, usually from the nephrotic syndrome. 
Given this patient's medical history, the cause of his pleural effusion is most
likely congestive heart failure. 

Pleural effusions are accumulations of fluid within the pleural space. They have
multiple causes and usually are classified as transudates or exudates. Detection is by
physical examination and chest x-ray; thoracentesis and pleural fluid analysis are
often required to determine cause. Asymptomatic transudates require no treatment.
Symptomatic transudates and almost all exudates require thoracentesis, chest tube
drainage, pleurodesis, and/or pleurectomy.

Exudative effusions are caused by local processes leading to increased capillary


permeability resulting in exudation of fluid, protein, cells, and other serum
constituents. Causes are numerous, the most common being pneumonia, malignancy,
pulmonary embolism, viral infection, and TB.

Question #72
QID: 5624
Topic: Pleural Effusion
Subject: Medicine

A 60-year-old male with a right-sided pleural effusion undergoes thoracentesis.


Analysis of the pleural fluid reveals a protein level of 2.0 g/dL and an LDH level of
70 U/L. His serum protein level is 7.0 g/dL (N 6.0-8.3) and his serum LDH level is
200 U/L (N 100-105). Based on these findings, which one of the following is the most
likely diagnosis?
a)  Heart failure
b)  Pulmonary embolism
c)  Tuberculous pleurisy
d)  Malignancy
e)  Bacterial pneumonia

The correct answer is a)

Explanation:
Pleural effusions may be exudates or transudates. The distinction is important for an
accurate diagnosis and to help determine what further evaluations may be necessary.
Light’s criteria use ratios of fluid/serum values for protein and LDH. Pleural
fluid/serum ratios greater than 0.6 for LDH and 0.5 for protein are indicative of
exudates. In the scenario presented, both ratios are approximately 0.3; therefore, the
fluid is a transudate.

The list of causes for transudates is much shorter than for exudates. The vast majority
of transudates are due to heart failure, with cirrhosis being the next most common
cause. Once there is reasonable certainty that the fluid is a transudate, additional
studies usually are not necessary. The other conditions listed result in exudative
pleural effusions.

Question #73
QID: 940
Topic: Hyperventilation
Subject: Medicine

An 18-year-old woman previously in good health seeks help at an emergency room


for lightheadedness, headaches and nausea. She appears anxious and is tremulous,
sweating, and breathing heavily. These symptoms appeared suddenly 20 min before
the patient’s arrival to the ED. While waiting to see a physician, she begins to
complain of tingling around her mouth and in her tongue.

Once life-threatening conditions are eliminated, which one of the following would be
your first management step for this patient?
a)  Reassurance and an explanation of how hyperventilation produces the symptoms
b)  Order immediate intravenous infusion of a glucose solution
c)  Order a urine drug screen
d)  Give supplemental oxygen by mask
e)  Administer 2 mg of sublingual lorazepam (Ativan)

The correct answer is a)

Explanation:
The symptoms experienced by this patient are classical symptoms of hyperventilation,
which commonly is associated with panic disorder and other anxiety states.
Hyperventilation causes a drop in blood CO and alkalosis, which in turn causes a
2

decrease in the ionized fraction of the serum calcium and constriction of the cerebral
vessels. Dizziness, lightheadedness, and feelings of derealization follow the cerebral
hypoxia. The lower ionized calcium level causes signs of tetany, such as painful
muscle spasms in the hands, perioral tingling, and paresthesias.

Once life-threatening conditions are eliminated, simple reassurance and an


explanation of how hyperventilation produces the patient’s symptoms are usually
sufficient to terminate the episode. Provoking the symptoms by having the patient
voluntarily hyperventilate for 3-4 minutes often convinces the patient of the diagnosis,
but it is time-consuming and may be ineffective.

Question #74
QID: 11243
Topic: Hyperventilation
Subject: Medicine

A 26-year-old woman was brought to hospital with sudden onset breathlessness,


agitation, central chest pain, and dizziness. For the last two weeks she has been on
flurazepam for insomnia. She loves pigeons and she tends to a couple of them at
home.
On examination, her heart rate was 97 beats per minute, respiratory rate 26/minute,
blood pressure 124/82 mmHg, and body temperature 36.9°C. Physical examination
was normal. Myocardial ischemia was excluded with appropriate work-up. Complete
blood count was normal apart from a hemoglobin level of 110g/L. Blood gas analysis
was as follows:

pH: 7.62
PaO2: 118 mmHg
PaCO2: 20 mmHg
[HCO3-]: 20 mmol/L

Which of the following is the most likely cause of the symptoms and blood gas results
in this patient?

a)  Acute Bird Fancier Pneumonitis


b)  Hyperventilation syndrome
c)  Anemia
d)  Panic disorder
e)  Flurazepam ingestion

The correct answer is b)

Explanation:

Hyperventilation syndrome is typically a disease of young women. It usually presents


with tachypnea and hyperpnoea, which may be interpreted as breathlessness, atypical
chest pain, agitation, dizziness, and palpitations. Physical examination shows
tachypnea and sometimes wheezes. If severe, carpopedal spasm might develop. Blood
gas analysis of patients with hyperventilation syndrome usually shows respiratory
alkalosis. Our patient showed most of these features and hyperventilation syndrome
(choice B) is thus, the most likely diagnosis in this patient.

> Acute Bird Fancier Pneumonitis (choice A) is a rather unlikely cause of symptoms,
and blood gas results in this patient despite the history of pigeon breeding. Acute Bird
Fancier Pneumonitis typically presents with breathlessness, fever with chills, cough,
and malaise. These symptoms develop 4 to 6 hour following exposure. Physical
examination of patients with this disease usually shows diffuse crackles over the
lower lung fields. When patients with Bird Fancier Pneumonitis or any other form of
interstitial lung disease have low PaCO2, PaO2 will also be low due to ventilation
perfusion mismatch. This mismatch causes hypoxemia (low paO2) which stimulates
ventilation washing out CO2. Our patient did not show most of these features and
Bird Fancier Pneumonitis is thus, an unlikely in this patient.
> Anemia (choice C) is an unlikely cause of symptoms, and blood gas results in this
patient. Only severe anemia (hemoglobin below 80g/L) can stimulate ventilation and
result in respiratory alkalosis. Because of their high blood flow, the peripheral
chemoreceptors cover their oxygen need from the dissolve fraction of oxygen in
blood, which primary depends on PaO2 rather than hemoglobin level. Anemia in our
patient is mild and is thus, unlikely to explain symptoms in this patient.
> Panic attack (choice D) is a discrete period of intense fear or discomfort in which
four of 12 criteria should prevail. For the diagnosis of panic disorder to be made, one
attack should be followed by one month or more of persistent concern about having
additional attacks or a significant change in behaviour related to the attacks.
Although, panic attacks can cause hyperventilation, one cannot make the diagnosis of
panic disorder in this patient.
> Like other benzodiazepines, flurazepam (choice E) is used in the treatment of
hyperventilation syndrome because it can alleviate anxiety and suppress ventilation.

In young patients with normal physical examination and absence of hypoxemia,


hyperventilation is most likely due to hyperventilation syndrome.

Question #75
QID: 968
Topic: Theophylline Toxicity
Subject: Medicine

You are called to see an elderly hospitalized patient with mild obstructive lung
disease who was roused one morning with irritability, restlessness and muscle
twitching.

Which of the following drug orders is most likely associated with this episode?  

a)  Trimethoprim-sulfamethoxazole
b)  Hydrochlorothiazide
c)  Flurazepam
d)  Theophylline
e)  Senna

The correct answer is d)

Explanation:
Manifestation of Theophylline Acute Toxicity:

Theophylline has a low therapeutic index. Theophylline toxicity is most likely to


occur when serum concentrations exceed 20 mcg/mL and becomes progressively
more severe at higher serum concentrations. Tachycardia, in the absence of hypoxia,
fever, or administration of sympathomimetic drugs, may be an indication of
theophylline toxicity. Anorexia, nausea and occasional vomiting, diarrhea, insomnia,
irritability, restlessness, and headache commonly occur. The distinguishing symptoms
of toxicity may include agitated maniacal behavior, frequent vomiting, extreme thirst,
slight fever, tinnitus, palpitation, and arrhythmias. Patients may experience delirium,
muscle twitching, severe dehydration, albuminuria, emesis of a "coffee ground"
material, hyperthermia, and profuse diaphoresis. Seizures may occur even without
other preceding symptoms of toxicity and often result in death.

Question #76
QID: 1184
Topic: Mechanical Ventilation
Subject: Medicine

A 75-year-old man with a history of emphysema presents to his physician after he


develops the acute onset of fevers, rigors, and a cough productive of green sputum.
His temperature is 38.3oC (100.9 F), blood pressure is 155/90 mmHg, pulse is 90/min,
and respirations are 34/min. He weighs 80 kg. An arterial blood gas reveals a pH of
7.20, a pCO2 of 60 mmHg, and a pO2 of 52 mmHg. Over the next 2 hours, he becomes
increasingly tachypneic, and his pCO2 rises to 74 mmHg. The decision is made to
intubate him at that point.

Which of the following settings would be most appropriate for his tidal volume on the
respirator?  

a)  400 mL/breath
b)  640 mL/breath
c)  1040 mL/breath
d)  1200 mL/breath
e)   1600 ml/breath

The correct answer is b)

Explanation:
This patient presents with COPD exacerbation.
Appropriate initial ventilator settings would be volume assist control, rate 10 to 12
breaths/min; tidal volume, 8 mL/kg; positive end-expiratory pressure (PEEP) of 0 to 5
cm H2O; and an adequate FiO2 to keep the hemoglobin saturation near 92%. These
patients require high flows (peak flow 75 to 90 L/min) to allow for an inspiratory to
expiratory ratio of 1:4 if possible. The initial settings should be adjusted by assessing
the patient’s comfort level and synchrony with the ventilator and by arterial blood gas
analysis.

> Giving a lower tidal volume will yield hypoventilation and be insufficient to
eliminate pCO . 2

> Providing a tidal volume greater than 10 mL/kg increases the risk of pneumothorax,
particularly in a patient with longstanding emphysema who may have thin-walled
alveoli.

Question #77
QID: 1443
Topic: Bronchospasm
Subject: Medicine

A 39-year-old automobile mechanic presents because of the new onset of wheezing.


His medications are propranolol, enalapril, hydrochlorothiazide, ranitidine and
occasional pseudoephedrine for symptoms of allergic rhinitis. On physical
examination, he appears comfortable. His blood pressure is 134/88 mm Hg, pulse is
68/min, and respirations are 18/min. On lung examination, soft expiratory wheezes
are heard throughout both lung fields.

Which of the following medications is most likely contributing to his wheezing?  

a)  Enalapril
b)  Hydrochlorothiazide
c)  Propranolol
d)  Pseudoephedrine
e)  Ranitidine

The correct answer is c)

Explanation:
Propranolol, like other nonspecific beta blockers, may cause bronchospasm by
blocking the beta receptors in the bronchial tree. Beta stimulation in the lungs
produces bronchodilation, and its blockade leads to bronchoconstriction. In fact,
propranolol is contraindicated in patients with known asthma or chronic obstructive
pulmonary disease (COPD).

Question #78
QID: 1628
Topic: Adverse Drug Effect
Subject: Medicine

A 40 year old black female presents to your office complaining of a persistent dry
cough for the last 3 months. This seemed to start with a “bad chest cold”. She went to
an urgent-care facility 6 weeks ago and the physician prescribed albuterol (Proventil,
Ventolin) by metered-dose inhaler empirically for presumed reactive airways disease.
This did not help. Now she also reports dyspnea on exertion that is noticeable when
walking uphill. She has been taking nitrofurantoin (Macrodantin) for chronic urinary
tract infections, but has an otherwise negative history. She works as a legal secretary.

On examination, she is tachypneic. There is no cyanosis or clubbing. Her lungs are


clear. Her height is 160 cm (63 in) and her weight is 60 kg (132 lb). Office pulmonary
function tests reveal a forced vital capacity (FVC) of 1.4 L (average 3.3) and a 1-
second forced expiratory volume (FEV1) of 1.6 L (average 2.8). An inhaled
bronchodilator produces no improvement in these numbers.

Which one of the following is the most likely cause of her problem?  
a)  Chronic asthma
b)  Persistent coughing resulting from a viral respiratory illness
c)  Cardiac disease
d)  Interstitial lung disease

The correct answer is d)

Explanation:
This patient has a reduced FVC with an FEV1/FVC ratio of 1.14%. This is consistent
with moderately severe pulmonary restriction. Most likely the patient has chronic
interstitial restrictive lung disease. Nitrofurantoin can cause this picture, usually after
continuous treatment for 6 or more months, and pulmonary function may be impaired
permanently. A wide variety of additional causes have been described including
noxious gases, pulmonary hypersensitivities, neoplasia, and systemic diseases (e.g.,
sarcoidosis). Management includes avoidance of the offending agent or treatment of
the underlying condition.

Question #79
QID: 1629
Topic: Acute Mountain Sickness
Subject: Medicine

A 38 year old white male wants to go to the mountains to ski. In the past he has
experienced moderate symptoms of acute mountain sickness (AMS), including
headache, nausea, shortness of breath, and sleep disturbance. He has been otherwise
healthy.

Which one of the following has been shown to be helpful in minimizing or preventing
the symptoms of AMS?  

a)  Furosemide (Lasix)
b)  Erythromycin
c)  Acetazolamide (Diamox)
d)  Beta-Blockers
e)  Fluid restriction

The correct answer is c)

Explanation:
Acute mountain sickness (AMS) is a clinical syndrome which may affect as many as
12%-67% of persons ascending to altitudes of 8000 feet or greater. While a standard
definition of AMS does not exist, persons having three or more of the following
symptoms may be considered to have AMS: headache, nausea, vomiting, sleep
disturbance, anorexia, fatigue, or dyspnea. Gradual ascent is recommended to allow
acclimatization.
Measures which may help minimize symptoms include avoidance of alcohol,
increased fluid intake, and a high-carbohydrate diet. Fluid restriction and diuretics
should be avoided because of the diuresis associated with acclimatization, which may
by itself cause dehydration. Antibiotics are of no benefit. beta-Blockers would be
harmful, by interfering with the physiologic responses of tachycardia and increased
cardiac output at higher altitudes.

Acetazolamide speeds the process of acclimatization. The drug is a carbonic


anhydrase inhibitor which results in a renal bicarbonate diuresis and metabolic
acidosis, thereby increasing ventilation and arterial oxygenation. The respiratory
stimulation is particularly important during sleep, when it reduces the severe
hypoxemia caused by periodic breathing. The drug also lowers cerebrospinal fluid
volume and pressure, which may play an additional role in it s therapeutic and
prophylactic utility.

Question #80
QID: 3258
Topic: Acute Mountain Sickness
Subject: Medicine

A 28-year-old male visits your office because he is planning a ski trip. You practice in
a coastal area, and he plans to be at an altitude of 14,500 feet. On a previous ski trip to
the same altitude he experienced symptoms of headache, poor sleep, anorexia, fatigue,
nausea, and vomiting. He asks you what he can do to prevent these symptoms on his
upcoming trip.

Which one of the following would you recommend?  

a)  Caffeine avoidance
b)  Caffeine tablets
c)  Furosemide (Lasix)
d)  Acetazolamide (Diamox)
e)  Fluid restriction

The correct answer is d)

Explanation:
This patient experienced acute mountain sickness (AMS), which is the most common
altitude illness. It occurs in 40%-50% of persons from low altitudes who ascend to
14,000 feet. The onset can occur within 8 to 96 hours of arrival at altitudes above
8000 feet, although the altitudes at which symptoms begin vary significantly. AMS is
a clinical diagnosis, with the most common symptoms consisting of headache, poor
sleep, anorexia, fatigue, nausea, and vomiting. Slow ascent is the best way to avoid
AMS. Adequate hydration may be helpful. Acetazolamide and dexamethasone help
prevent or mitigate the symptoms of AMS. Individuals who have had AMS in the past
should probably be treated prophylactically with acetazolamide.
Acetazolamide is a carbonic anhydrase inhibitor that causes a hyperchloremic
metabolic acidosis through the loss of bicarbonate, sodium, and potassium in the
urine. Respiration is stimulated by the acidosis, which leads to a compensatory
respiratory alkalosis. Pretreatment with this agent mimics the acclimated state of acid-
base balance, so that during the first day of altitude exposure, subjects taking this drug
have values for pH, partial pressure of arterial carbon dioxide, and minute ventilation
that are not typically observed until day 5 in control subjects.

Question #81
QID: 5508
Topic: Acute Mountain Sickness
Subject: Medicine

A 47-year-old male who lives at sea level attempts to climb a mountain. On the first
day he ascends to 3400 m (11,000 ft). The next morning he complains of headache,
nausea, dizziness, and fatigue, but as he continues the climb to the summit he
becomes ataxic and confused. Which one of the following is the treatment of choice?
a)  Administration of oxygen and immediate descent
b)  Dexamethasone, 8 mg intramuscularly
c)  Acetazolamide (Diamox), 250 mg twice a day
d)  Nifedipine (Procardia), 10 mg immediately, followed by 30 mg in 12 hours
e)  Helicopter delivery of a portable hyperbaric chamber

The correct answer is a)

Explanation:
The patient described initially showed signs of acute mountain sickness. These
include headache in an unacclimatized person who recently arrived at an elevation
>2500 m (8200 ft), plus the presence of one or more of the following: anorexia,
nausea, vomiting, insomnia, dizziness, or fatigue. The patient’s condition then
deteriorated to high-altitude cerebral edema, defined as the onset of ataxia and/or
altered consciousness in someone with acute mountain sickness. The management of
choice is a combination of descent and supplemental oxygen. Often, a descent of only
500-1000 m (1600-3300 ft) will lead to resolution of acute mountain sickness.
Simulated descent with a portable hyperbaric chamber also is effective, but descent
should not be delayed while awaiting helicopter delivery. If descent and/or
administration of oxygen is not possible, medical therapy with dexamethasone and/or
acetazolamide may reduce the severity of symptoms. Nifedipine has also been shown
to be helpful in cases of high-altitude pulmonary edema where descent and/or
supplemental oxygen is unavailable.

Question #82
QID: 1668
Topic: Lung Abscess
Subject: Medicine

A 25-year-old white male who has a poorly controlled major seizure disorder and a 6-
week history of recurrent fever, anorexia, and persistent, productive coughing visits
your office. On physical examination he is noted to have a temperature of 38.3°C
(101.0°F), a respiratory rate of 16/min, gingival hyperplasia, and a fetid odor to his
breath. Auscultation of the lungs reveals rales in the mid-portion of the right lung
posteriorly.

Which one of the following is most likely to be found on a chest radiograph?  

a)  Sarcoidosis
b)  Miliary calcifications
c)  A lung abscess
d)  A right hilar mass
e)  A right pleural effusion

The correct answer is c)

Explanation:
Anaerobic lung abscesses are most often found in a person predisposed to aspiration
who complains of a productive cough associated with fever, anorexia, and weakness.
Physical examination usually reveals poor dental hygiene, a fetid odor to the breath
and sputum, rales, and pulmonary findings consistent with consolidation.

> Patients who have sarcoidosis (choice A) usually do not have a productive cough
and have bilateral physical findings.
> A persistent productive cough is not a striking finding in disseminated tuberculosis,
which would be suggested by miliary calcifications (choice B) on a chest film.
> The clinical presentation and physical findings are not consistent with a simple
mass in the right hilum (choice D) nor with a right pleural effusion (choice E).

Question #83
QID: 2979
Topic: Lung Abscess
Subject: Medicine

This question is no longer available.


Thank you for your understanding and cooperation.

The correct answer is

Explanation:
Question #84
QID: 10844
Topic: Lung Abscess
Subject: Medicine

A 52-year-old male presents with productive cough and generalized malaise for the
last 2 weeks. Three weeks ago he underwent surgery at your hospital.
Physical examination reveals diminished breath sounds in the right mid-lung field.
Chest X-ray shows an infiltrative shadow with a cavity formation in the right mid-
lung field. Assuming the etiology of his condition is the known most common cause,
which of the following is the most effective treatment?

a)  Clindamycin
b)  Trimethoprim-Sulfamethoxazole
c)  Vancomycin
d)  Voriconazole
e)  Metronidazole

The correct answer is a)

Explanation:
This patient presents with some non-specific respiratory system symptoms but the
physical examination and the findings on chest X-ray are suggestive of a lung abscess.
It is defined as necrosis of the pulmonary tissue and formation of cavities containing
necrotic debris or fluid caused by microbial infection. It may be primary or secondary.
A primary lung abscess occurs when one or two cavities with air-fluid levels form in
the lung parenchyma as the result of an aspiration of pathogen-laden secretion. A
secondary lung abscess develops from predisposing conditions, such as congenital
lung abnormalities, obstructing neoplasm, a foreign body, and bronchiectasis.
Primary lung abscesses are more common than secondary lung abscesses and
aspiration accounts for 80- 90% of primary lung abscesses and particularly so in post-
operative patients. Given this etiology, majority of lung abscesses are caused by
anaerobic bacteria. Assuming this to be the etiology of this patient’s condition, he
should be treated with clindamycin (choice A). It is a linconsamide antibiotic that can
cover both anaerobic bacteria as well as methicillin-resistant staphylococcus.

> Trimethoprim-sulfamethoxazole (choice B) is a sulfonamide that can be used to


treat gram-negative and gram-positive bacteria associated infections but it is
ineffective in the treatment of lung abscess caused by anaerobic bacteria.
> Vancomycin (choice C) is used to treat penicillin-resistant Staphylococcus
aureus; this is not the most common infectious agent involved in lung abscesses.
> Voriconazole (choice D) would be appropriate to treat invasive aspergillosis; it is
not the treatment to use for anaerobic bacteria-associated lung abscesses.
> Metronidazole (choice E) is commonly used to treat anaerobic bacteria and protozoa
infections; however, its use in the treatment of lung abscesses has been disappointing
as it is reported to have 50% failure rate. Therefore, metronidazole would not be the
most effective drug for the management of this patient’s presenting condition.

Key point:
Aspiration is the most common cause of lung abscess in post-operative patients. Since
majority of aspiration-induced lung abscesses infectious agents are anaerobic bacteria,
the most effective treatment is clindamycin.

Question #85
QID: 2891
Topic: BiPAP
Subject: Medicine

Bilevel positive airway pressure ventilation (BiPAP) has been shown to reduce
morbidity and mortality from which one of the following?  

a)  Acute respiratory distress syndrome 


b)  Respiratory failure associated with severe pneumonia 
c)  Sepsis 
d)  Pneumothorax 
e)  Exacerbation of COPD

The correct answer is e)

Explanation:
Bilevel positive airway pressure (BiPAP) is beneficial in the treatment of patients
with progressive respiratory acidosis, impending respiratory failure, or progressive
fatigue associated with increased work of breathing. In patients with acute
exacerbations of COPD, it not only improves ventilation while decreasing pCO2
levels, but may well be the key to avoiding intubation. BiPAP has been shown to be
deleterious in the treatment of respiratory failure associated with sepsis, pneumonia,
acute respiratory distress syndrome, or pneumothorax.

Question #86
QID: 3224
Topic: Cough
Subject: Medicine

The most common cause of chronic cough in adults is:  

a)  gastroesophageal reflux disease


b)  asthma
c)  upper airway cough syndrome (postnasal drip)
d)  nonasthmatic eosinophilic bronchitis
e)  ACE inhibitor use

The correct answer is c)


Explanation:
Causes of chronic cough in the adult patient are often somewhat obscure, as the onset
may be vague and there are no consistently reliable defining characteristics. Further
complicating this diagnostic challenge is the possibility that the cough may be due to
multiple concurrent conditions that must each be addressed before the cough will
resolve. Each of the options listed may cause chronic cough and must be considered.

In order, the three most common causes of chronic cough are upper airway cough
syndrome (UACS), asthma, and gastroesophageal reflux disease. Nonasthmatic
eosinophilic bronchitis is frequent enough to warrant consideration as well. A 2001
decision analysis provides support for the strategy of treating chronic cough
empirically. Since the most common cause for chronic cough is UACS (with an
assumed prevalence of 44%) due to rhinosinus conditions, an empiric trial of a first-
generation antihistamine/decongestant is a reasonable option. Further treatment
choices are based on the response to this regimen.

Question #87
QID: 3250
Topic: Hypersensitivity Pneumonitis
Subject: Medicine

A 40-year-old white male was seen 4 weeks ago for a sudden onset of cough and
shortness of breath. At that visit his oxygen saturation was 92%, but his examination
and a chest radiograph were normal. You prescribed azithromycin (Zithromax) and an
albuterol inhaler. Ten days later he was feeling well and his oxygen saturation was
97%. Today, he returns to the office with a dry cough and shortness of breath.

On examination he has rare inspiratory rales that clear with deep breaths, and he has
an oxygen saturation of 86%. A chest film and a D-dimer test are normal. Pulmonary
function tests show significant restriction that improves only minimally with
albuterol. He has not been exposed to anyone with a similar illness, has no history of
asthma, and has no smoking history or occupational exposure. However, he reports
that 2 months ago his home was flooded after a heavy rain, and he has been tearing
out carpeting that was ruined by the flood.

Which one of the following is the most likely diagnosis?  

a)  Persistent asthma with acute exacerbations


b)  Legionnaires’ disease
c)  Pulmonary embolism, with lupus antibody as the most likely cause of the negative
D-dimer test
d)  Hypersensitivity pneumonitis

The correct answer is d)


Explanation:
Hypersensitivity pneumonitis can present in acute, subacute, or chronic forms. The
case described includes two episodes of the acute form. The patient was exposed to
mold antigens in his flooded home. Within 4-8 hours of exposure, chills, cough, and
shortness of breath will be noted, and at times will be dramatic. A chest film can be
normal, even with significant hypoxia. Pulmonary function tests will show restrictive
changes, as compared to the reversible obstructive changes of acute asthma. Blood
tests often show an elevated erythrocyte sedimentation rate. Serum IgG tests for the
probable antigen confirm the diagnosis.

Symptoms resolve over several days, but will suddenly and violently recur with
repeated exposure to the offending antigen. The subacute form of hypersensitivity
pneumonitis begins gradually over weeks or months, causing a cough and increasing
shortness of breath. The chronic form develops over years of exposure, causing
fibrotic changes to the lungs that will be evident on radiographs, as well as chronic
rales on auscultation.

Asthma would be an unlikely diagnosis in this case, with the pulmonary function tests
showing restrictive changes rather than obstructive changes, and little improvement
with albuterol. Also, the lack of a previous history of asthma makes it less likely.
Legionnaires’ disease is always possible, but is unlikely in this case given the sudden
onset, quick recovery over several days, and sudden recurrence. Pulmonary embolism
is ruled out by the negative D-dimer test.

Question #88
QID: 3312
Topic: Hemoptysis
Subject: Medicine

The most common cause of hemoptysis seen in the primary care setting is:  

a)  pulmonary embolism
b)  lower respiratory tract infection
c)  lung cancer
d)  heart failure
e)  asthma

The correct answer is b)

Explanation:
Infection of the lower respiratory tract causes well over half of all cases of
hemoptysis, with bronchitis implicated more often than pneumonia. Lung cancer
should always be considered, as it may account for up to a quarter of hemoptysis
cases. Pulmonary embolism is a much less common cause.
Question #89
QID: 3326
Topic: Death Rattle Treatment
Subject: Medicine

Which one of the following palliative measures is appropriate for a patient dying of
end-stage Alzheimer’s disease who is experiencing the “death rattle” in the final hours
of life?  

a)  Subcutaneous administration of fluids


b)  Rectal administration of fluids
c)  Glycopyrrolate (Robinul)
d)  Bethanechol (Urecholine)
e)  Rivastigmine (Exelon)

The correct answer is c)

Explanation:
Anticholinergic medications, such as glycopyrrolate, may be used effectively to dry
the secretions that cause the “death rattle” in terminally ill patients.

>Bethanechol and rivastigmine are both cholinergic, which has the opposite effect.
>Administration of fluids by hypodermoclysis or proctoclysis would increase
respiratory secretions, thereby increasing the symptoms.

Question #90
QID: 5463
Topic: Exercise-induced Bronchoconstriction
Subject: Medicine

A 22-year-old competitive cross-country skier presents with a complaint of not being


able to perform as well as she expects. She has been training hard, but says she seems
to get short of breath more quickly than she should. She also coughs frequently while
exercising. A review of systems is otherwise negative. Her family history is negative
for cardiac or pulmonary diseases. Her physical examination is completely normal,
and pulmonary function tests obtained before and after bronchodilator use are normal.

After you discuss your findings with the patient, she acknowledges that her
expectations may be too high, but can think of no other cause for her problem.

Which one of the following would be the next reasonable step?


a)  An echocardiogram to look for cardiomyopathy or valvular dysfunction
b)  Counseling regarding competition stress and athlete burnout syndrome
c)  A sports medicine consultation to evaluate her training regimen
d)  A trial of inhaled albuterol (Proventil) for exercise-induced bronchospasm
The correct answer is d)

Explanation:
Exercise-induced bronchoconstriction (EIB) is a very common and underdiagnosed
condition in athletes. It is defined as a 10% lowering of FEV1 when challenged with
exercise. The exercise required to cause bronchoconstriction is 5-8 minutes at 80% of
maximal oxygen consumption.

EIB is much more common in high-ventilation sports, such as track and cross-country
skiing. It is also more common in winter sports, because of the inspiration of cold, dry
air. In some studies the incidence among cross-country skiers is as high as 50%, and
40% of those who have positive tests for bronchospasm are unaware of the problem.
A physical examination, as well as pulmonary function tests at rest and before and
after bronchodilators, will be normal unless there is underlying asthma. Among
athletes with EIB, 10% will not have asthma.

Bronchoprovocative testing can be ordered, but if it is not available a trial with an


albuterol inhaler is reasonable. Cardiomyopathy or valvular dysfunction not found
during the physical examination is possible, but much less likely. Psychological
stresses are also a possible etiology, but should not receive undue attention, especially
when simple questioning is not productive and more likely diagnoses have not been
ruled out. Poor training methods are also possible, but in a competitive athlete this is
not the most likely cause.

Question #91
QID: 5585
Topic: Asbestosis
Subject: Medicine

A 60-year-old male presents to the urgent care center with a fever and a productive
cough. He has a 40 pack/year history of cigarette smoking. In addition to lobar
pneumonia on a chest radiograph, there is an incidental finding of bilateral pleural
plaques. Which one of the following is the most likely cause of this finding?
a)  Coal dust
b)  Silicon dust
c)  Asbestos
d)  Vinyl chloride
e)  Radon

The correct answer is c)

Explanation:
Development of pleural plaques is the most common pathologic pulmonary response
to asbestos inhalation. Over time, collagen is deposited in the pleura and may calcify.
Most plaques are asymptomatic, and there is no evidence that plaques transform into
malignant lesions. Plaques occur in approximately 50% of persons with heavy and
prolonged exposure to asbestos and, therefore, are a marker of asbestos exposure.
This should alert you to follow the patient for development of more serious asbestos-
related diseases (e.g., lung cancer and mesothelioma).

>Although the other substances listed are associated with pulmonary diseases (coal
dust and silicon dust with pneumoconiosis, and vinyl chloride and radon with lung
cancer), none is associated with pleural plaques as found in this patient.

Question #92
QID: 8762
Topic: Asbestosis
Subject: Medicine

Which one of the following is not typically seen in asbestosis?


a)  Increased risk of bronchogenic cancer
b)  Pleural thickening and calcification
c)  Interstitial fibrosis
d)  Obstructive pattern on pulmonary function tests
e)   Finger clubbing

The correct answer is d)

Explanation:
Asbestosis is a chronic and restrictive lung disease. Total lung capacity is reduced in
asbestosis as in other restrictive disorders. It is caused by inhalation of asbestos
particles over prolonged periods of time. 
There is typically a prolonged interval (20-30 yrs) between exposure and clinical
manifestations of disease.

> signs and symptoms:


• insidious onset
• SOB on exertion usually first symptom with increased dyspnea as disease progresses

• cough: paroxysmal, non-productive


• fine end-respiratory crackles (increased at bases)
• clubbing (much more likely in asbestosis than silicosis or coal workers'
pneumoconioses), edema, jugular venous distention

> investigations: CXR:


• lower > upper lobe
• early: fibrosis with linear streaking
• later: cysts and honeycombing
• asbestos exposure can also cause pleural and diaphragmatic plaques (±
calcification), pleural effusion, round atelectasis

> Asbestos exposure increases risk of bronchogenic cancer and


malignant mesothelioma.

Question #93
QID: 11016
Topic: Asbestosis
Subject: Medicine

A 49-year-old male presents to your department with complaints of increasing


shortness of breath and non-productive cough for the last 3 months. He has only tried
over the counter antitussive medications, which have helped only a little with
coughing but not with shortness of breath. He denies weight loss, fever, and night
sweats. Velcro-like crackles are heard posterolaterally at the lung bases. He has been
working in shipbuilding for the last 23 years and denies smoking or drinking alcohol.
What should you be looking for in the next step of managing this patient’s condition?

a)  Asbestos bodies on lung-tissue biopsy


b)  Pleural plaques in the diaphragmatic pleura on X-ray
c)  Obstructive-pattern on pulmonary function studies
d)  Peripheral lung lesions on chest X-ray
e)  Positive antinuclear antibodies with high titer

The correct answer is b)

Explanation:
This patient’s presentation with dyspnea and non-productive cough and findings on
physical examination of velcro-like crackles posterolaterally (fine crackles) at the
lung bases are suggestive of asbestosis. This is even more likely given the fact he has
worked in shipyards for more than 20 years, as this could have exposed him to
asbestos. Pulmonary function studies and chest X-ray should be done as initial
studies. In patients with asbestosis pulmonary function studies reveal restrictive lung
disease and pleural plaques in the diaphragmatic pleura on X-ray are considered a
reliable indicator of asbestos exposure, and in a patient with classic clinical
presentation (choice B).

> Asbestos bodies on lung-tissue biopsy (choice A) are commonly found but such an
invasive study should not be the next step in the management of this patient.
> Obstructive-pattern on pulmonary function studies (choice C) is incorrect.
Asbestosis is most likely to show a restrictive-pattern on pulmonary function studies.
> Peripheral lung lesions on chest X-ray (choice D) are characteristic of
adenocarcinoma. In patients with asbestos exposure the most concerning consequence
is malignant mesothelioma, which on X-ray is seen as obliteration of the diaphragm;
nodular thickening of the pleura; or sheetlike encasement of the pleura; or a
combination of all of these.
> Positive antinuclear antibodies with high titer (choice E) is incorrect. ANA lack
diagnostic specificity and are not useful in diagnosis

Key point:
Exposure to asbestos in shipyards is a risk factor for asbestosis, which presents with
dyspnea and non-productive cough and fine crackles posterolaterally at the lung
bases. Pulmonary function tests show restrictive lung disease and Chest X-ray shows
diaphragmatic pleural plaques.

Question #94
QID: 10246
Topic: COPD Exacerbation Management
Subject: Medicine

A 52-year-old female with a 60-pack-year history of cigarette smoking and known


COPD presents with a 1-week history of increasing purulent sputum production and
shortness of breath on exertion. Which one of the following is true regarding the
management of this problem?

a)  Antibiotics should be prescribed


b)  Intravenous corticosteroids are superior to oral corticosteroids
c)  Inhaled corticosteroids should be started or the dosage increased
d)  Levalbuterol (Xopenex) is superior to albuterol
e)  Acetylcysteine should be given if the patient is hospitalized

The correct answer is a)

Explanation:
Antibiotic use in moderately or severely ill patients with a COPD exacerbation
reduces the risk of treatment failure or death, and may also help patients with mild
exacerbations. Brief courses of systemic corticosteroids shorten hospital stays and
decrease treatment failures. Studies have not shown a difference between oral and
intravenous corticosteroids. Inhaled corticosteroids are not helpful in the management
of an acute exacerbation. Levalbuterol and albuterol have similar benefits and adverse
effects.

Acetylcysteine, a mucolytic agent, has not been shown to be helpful for routine
treatment of COPD exacerbations.

Question #95
QID: 10841
Topic: COPD Exacerbation Management
Subject: Medicine

A 47-year-old male presents to the ED with complaints of cough, shortness of breath,


and increasing purulent sputum over the past 3 days. He was diagnosed with COPD 6
years ago. Last spirometry results, 9 months ago, showed FEV of 55%. His
medications are tiotropium and albuterol.
On examination the patient is alert and oriented to person, place, and time; BP is
135/80 mmHg, pulse 100 bpm, respiratory rate 20/min, and temperature 37.8 oC.
Auscultation of the chest reveals widespread respiratory wheeze and inspiratory
coarse crackles in the left lung base. No cyanosis, accessory muscle use, and
peripheral edema are noted. Which of the following drugs will be part of the treatment
of this patient’s current condition?

a)  Formoterol
b)  Oral Prednisone
c)  Metronidazole
d)  Tiotropium
e)  Guaifenesin

The correct answer is b)

Explanation:
This patient has a history of COPD and his symptoms that have worsened the last 3
days with dyspnea, cough, and purulent sputum are suggestive of COPD exacerbation.
An exacerbation of COPD is defined as an event of acute onset in the natural course
of the disease characterized by a change in the patient’s baseline dyspnea, cough, and
sputum that is beyond normal day-to-day variations. The Global Initiative for Chronic
Obstructive Lung Disease (GOLD) classifies COPD into 4 categories:
mild with FEV ≥ 80% of predicted,
moderate with FEV≥ 50% but < 80%,
severe COPD with FEV <50% but ≥ 30%, and
very severe COPD with FEV < 30%.
Since this patient’s last FEV was 55%, he is in the moderate COPD category. Given
the patient’s moderate COPD history, absence of significant comorbidities, and alert
mental status, his exacerbation is considered mild.

His management should include short-acting bronchodilator therapy. Initially short


acting beta-2-agonists such as inhaled albuterol are give. If a prompt response to these
drugs does not occur, a short acting anticholinergic such as ipratropium can be given.
Oral or intravenous corticosteroids (choice B) are recommended and they shorten
recovery time, improve lung function (FEV1), arterial PaO 2, and risks of early relapse.

According to the Winnipeg criteria (also called Anthonisen criteria) antibiotics are
indicated in patients with at least two of the following: (1) dyspnea (2) sputum
production (3) sputum purulence. This patient has all the three and he should be
treated with antibiotics suitable for mild exacerbations (e.g amoxicillin, trimethoprim-
sulfamethoxazole, and azithromycin). Oxygen should be given to achieve adequate
levels of oxygenation (PaO2: 60 mmHg and SaO2 ≥90%).

> Formoterol (choice A) is incorrect. While formoterol, a long acting beta-2-agonist


has been shown to reduce exacerbation recurrence; it is not suitable for acute
bronchospasm relief and should not be commenced during an acute exacerbation
episode.
> Metronidazole (choice C) covers anaerobic bacteria and protozoa; it is not
appropriate as antibiotic therapy for this patient. The most common infectious agents
associated with COPD acute exacerbations are Haemophilus influenza, Streptococcus
pneumonia, and Moraxella catarrhalis.
> Tiotropium (choice D) is incorrect. Tiotropium is a long-acting inhaled
anticholinergic that is used in the management of stable COPD. The preferred
bronchodilator therapy for acute exacerbation is short-acting beta-2-agonists.
> Guaifenesin (choice E) is an antitussive medication. It is not recommended because
cough has a protective role in COPD as it allows sputum clearance, which prevents
accumulation of inflammatory mucus exudates in the small airways.

Key point:
The management of chronic bronchitis exacerbation includes short acting
bronchodilator therapy, systemic corticosteroids, antibiotic therapy, and oxygen
therapy to achieve PaO2: 60 mmHg or SaO2 ≥90%.

Question #96
QID: 10310
Topic: Embolism Work Up
Subject: Medicine

A 58-year-old male presents with a several-day history of shortness of breath with


exertion, along with pleuritic chest pain. His symptoms started soon after he returned
from a vacation in South America. He has a history of deep-vein thrombosis (DVT) in
his right leg after surgery several years ago, and also has a previous history of prostate
cancer. You suspect pulmonary embolism (PE).

Which one of the following is true regarding the evaluation of this patient?

a)  CT angiography would reliably either confirm or rule out PE


b)  A negative compression ultrasonographic scan markedly reduces the likelihood of
pulmonary embolism.
c)  Chest radiographs are normal in most cases of pulmonary embolism
d)  No further testing is needed if a D-dimer level is normal
e)  An elevated D-dimer level would confirm the diagnosis of PE

The correct answer is a)

Explanation:
This patient has a high clinical probability for pulmonary embolism (PE). About 40%
of patients with PE will have positive findings for deep-vein thrombosis in the lower
extremities on compression ultrasonography. A multidetector CT angiogram or
ventilation-perfusion lung scan should be the next test, as these are reliable to confirm
or rule out PE. V/Q scanning should be used only when CT scanning is not available
or if the patient has a contraindication to CT scanning or intravenous contrast
material.

> A negative ultrasonographic scan does not rule out DVT, because many DVTs
occur in areas that are inaccessible to ultrasonographic examination. In two thirds of
patients with pulmonary embolism, the site of DVT cannot be visualized with
ultrasonography.
> Chest radiographs are abnormal in most cases of pulmonary embolism, but the
findings are nonspecific.
> A normal D-dimer level reliably rules out the diagnosis of venous
thromboembolism in patients at low or moderate risk of pulmonary embolism, but the
negative predictive value of this test is low for high-probability patients.
> A positive D-dimer test does not confirm the diagnosis; it indicates the need for
further testing, and is thus not necessary for this patient.

Question #97
QID: 10887
Topic: Embolism Work Up
Subject: Medicine

A 58-year-old female with a history of diabetes and hypertension, who recently


underwent surgery at your hospital, complains of sudden severe chest pain. Her ECG
is shown below:

Which of the following is the best next study in the diagnosis of this condition?
a)  Cardiac enzyme studies
b)  Ventilation/Perfusion scan
c)  Conventional pulmonary angiography
d)  Stress echocardiography
e)  Coronary angiography

The correct answer is b)

Explanation:
This patient’s chest pain is a nonspecific complaint that suggests a number of possible
diagnoses; however, the patient’s ECG reveals an S1Q3T3 pattern, which is
suggestive of pulmonary embolism.
A simplified image is shown here: 
This is known as the McGinn-White sign and indicates acute right heart strain. It is
present in only 20% of pulmonary embolism cases and may be seen in other causes of
acute cor pulmonale such as pneumothorax and bronchospasm. When pulmonary
embolism is suspected the best initial study is ventilation/perfusion scan (choice B) or
CT pulmonary angiogram. The ECG is a poor diagnostic tool for pulmonary
embolism and its main utility is to exclude other potentially life-threatening
conditions such as a myocardial infarction. If the ventilation/perfusion scan is
positive, treatment can be initiated.

> Cardiac enzyme studies (choice A) would be most appropriate if a diagnosis of


myocardial infarction is considered. This patient’s ECG showing the S1Q3T3 pattern
is suggestive of pulmonary embolism.
> Conventional pulmonary angiography (choice C) is considered the gold-standard
study in the diagnosis of pulmonary embolism; however, it is an invasive test that
would only be done if a ventilation/perfusion scan or CT pulmonary angiogram is
negative in a patient highly suspected to have pulmonary embolism.
> Stress echocardiography (choice D) is mainly used to detect reduced coronary blood
flow, it is not the best study to diagnose pulmonary embolism.
> Coronary angiography (choice E) is used for diagnosing coronary artery disease; it
is not the best study in a patient suspected of pulmonary embolism.

Chest pain and the S1Q3T3 pattern on ECG are suggestive of pulmonary embolism.
Ventilation/ Perfusion scan or CT pulmonary angiogram are the best studies for
screening patients suspected of pulmonary embolism.

Question #98
QID: 10357
Topic: Pulmonary Nodule
Subject: Medicine

A 50-year-old male has a pre-employment chest radiograph showing a pulmonary


nodule. There are no previous studies available.

Which one of the following would raise the most suspicion that this is a malignant
lesion if found on the radiograph?

a)  The absence of calcification


b)  Location above the midline of the lung
c)  A diameter of 4 mm
d)  A solid appearance

The correct answer is a)

Explanation:
Pulmonary nodules are a common finding on routine studies, including plain chest
radiographs, and require evaluation. Radiographic features of benign nodules include
a diameter <5 mm, a smooth border, a solid appearance, concentric calcification, and
a doubling time of less than 1 month or more than 1 year.

Features of malignant nodules include a size >10 mm, an irregular border, a “ground
glass” appearance, either no calcification or an eccentric calcification, and a doubling
time of 1 month to 1 year.

Question #99
QID: 11123
Topic: Pulmonary Nodule
Subject: Medicine

A 52-year-old man presents to the clinic with history of cough of 2 weeks duration.
Cough is non-productive and non-spasmodic. He has received a course of amoxicillin
for five days with no relief. There is no history of fever or weight loss. He is non-
diabetic and smokes 2 packs a day for past 30 years.
There is no abnormality detected on physical examination. His complete blood count
is within normal limits and his erythrocyte sedimentation rate (ESR) is 45mm in first
hour by Westergren method. Chest CT scan shows a contrast enhancing solitary
nodule measuring 2cms in the inferior lobe of left lung. A chest radiograph done 2
years prior was reported to be normal.
What is the most appropriate next step in management of this patient?

a)  Repeat chest CT scan in 3 months


b)  Repeat chest CT scan in 6 months
c)  Biopsy of the lesion
d)  Mantoux test (Tuberculin test)
e)  Chest Magnetic Resonance Imaging scan

The correct answer is c)

Explanation:
Evaluation guidelines for solitary pulmonary nodule are based on nodule size and
patient’s risk factors for cancer. Nodule diameter of 8mm or more, ‘ground glass’
opacity, irregular borders and doubling of size in one month to one year, are pointers
to malignancy. History of smoking, old age, and extra-pulmonary malignancy are
considered as risk factor for cancer.
History of smoking and nodule of more than 8mm in the case described in the stem,
indicate high probability of malignancy. Hence, biopsy of the lesion (choice C) is the
most appropriate measure for accurate diagnosis.
Most solitary pulmonary nodules are benign lesions, such as infectious granulomas
and hamartomas. Malignant lesions include lung cancer, carcinoid tumor or
metastasis. Survival rates of lung cancer are very low but early diagnosis can improve
prognosis.

> Solitary nodule smaller than 8mm on a CT scan, in a person with high risk for
malignancy, can be followed by repeat CT scan at 3-6 months, (choice A and B) and
again at 12 and 24 months if the lesion remains stable. This is not relevant to the case
described as the lesion size is 20mm. Solitary nodule smaller than 8mm on a CT scan
in a person with low risk of malignancy, can be followed by repeat CT scan at 6-12
months. If there is no change in size at 12 months, no further follow up is indicated.
> Positive Mantoux test (choice D) is a pointer to the diagnosis of tuberculosis but it
does not rule out the possibility of neoplastic lung lesion.
> Chest MRI (choice E) is indicated when it is not possible to do CT chest for reasons
such as contrast allergy or renal insufficiency.

Any patient with evidence of large pulmonary nodule or nodule with notable growth
during follow-up should undergo biopsy for diagnosis.

Question #100
QID: 10880
Topic: Pulmonary Infiltrates
Subject: Medicine

A 34-year-old male comes for a follow up of his recent lower respiratory infection
treatment. Three weeks ago he had sought medical attention due to 5 days of cough,
fever, and night sweats. Chest X-ray had shown pulmonary infiltrates and he was
prescribed antibiotics with a presumed community-acquired pneumonia diagnosis.
Today he reports that he is still coughing and has been experiencing chest pain. His
past medical history is significant for ulcerative colitis, for which he has been
receiving treatment for the last 3 years. Chest x-ray repeat shows disseminated
pulmonary infiltrates. Laboratory studies reveal weakly acid-fast organisms. Taking
his past medical history, current presentation, and all investigation results into
consideration, you decide to treat him with trimethoprim-sulfamethoxazole. If his
condition improved within the following 3 months, you would assume that the most
probable culprit for causing his symptoms was:

a)  Pneumocystis jiroveci
b)  Nocardia asteroides
c)  Mycobacterium tuberculosis
d)  Coccidioides immitis
e)  Mycoplasma pneumonia

The correct answer is b)


Explanation:
This patient’s clinical picture is suggestive of pleuropulmonary nocardiosis caused by
Nocardia asteroides (choice B). It is an acute, subacute, or chronic infectious disease
that typically affects immunocompromised hosts, although cases in
immunocompetent patients also have been reported. This patient’s long history
of inflammatory bowel disease that was most probably treated with
immunosuppressive therapy is a typical association with this infection.
The disease may present with night sweats, fever, cough, and chest pain. It is
characterized by multiple pulmonary infiltrates on chest X-ray with a tendency to
central necrosis. Pulmonary infiltrates are often persistent for months as the disease is
refractory to antibiotherapy, just as described in this patient. While diagnosis may be
challenging, when it is suspected, the organisms can be visualized by a modified
Ziehl-Neelsen stain such as Fite-Faraco staining. It is a weakly acid-fast organism.
Despite the lack of controlled clinical trials and no general treatment guidelines for
norcadiosis, sulfonamides are considered the treatment of choice. Sulfadiazine or
trimethoprim-sulfamethoxazole should be given for a period of 6 months.

> Pneumocystis jiroveci (choice A) also could present with cough, fever, and


pulmonary infiltrates in immunocompromised patients. Furthermore, the treatment of
choice for this infection is trimethoprim-sulfamethoxazole. However, it is visualized
on silver-stain and not Ziehl-Neelsen stain, as it is not an acid-fast organism. 
> Mycobacterium tuberculosis (choice C) is classified among unusual organisms that
may cause persistent pulmonary infiltrates and just as seen in this patient, it can cause
night sweats, fever, cough and is an acid-fast organism visualized on Ziehl-Neelsen
stain. However, it is best treated with Isoniazid, rifampin, ethambutol, and
pyrazinamide.
> Coccidioides immitis (choice D) is a fungus, treated with ketoconazole.
> Mycoplasma pneumonia (choice E) would probably have been cleared with
antibiotics given for community acquired pneumonia. It is not an acid-fast organism.

Key point:
Pleuropulmonary nocardiosis is associated with persistent pulmonary infiltrates and
may be refractory to antibiotherapy. It is a weakly acid-fast organism and the first line
treatment is sulfonamides (either sulfadiazine or trimethoprim-sulfamethoxazole).

Question #101
QID: 11239
Topic: Goodpasture syndrome
Subject: Medicine

A 28-year-old male presents to your department with complaints of fever, cough, and
hemoptysis for the last 3 days. He has also noted knee joints pain, swelling of the
ankles and feet, and some changes with his urine color. He denies wheezing, sore
throat, and a runny nose. He had a respiratory tract infection 4 weeks ago. He has
smoked 1 pack of cigarettes a day for the last 3 years.
His vital signs are BP 160/100 mmHg, temperature 38.9°C, pulse 90 bpm,
respirations 23/min. Physical examination reveals inspiratory crackles, tenderness on
palpation of the knees, and pedal edema. No paranasal sinuses tenderness, nasal
lesions, and skin granulomas are noted. Which of the following findings are most
likely to be observed in this patient?
a)  Anti-double stranded DNA antibodies
b)  Eosinophilia
c)  Cytoplasmic antineutrophil cytoplasmic antibodies
d)  Anti-glomerular basement membrane antibodies
e)  Anti-citrullinated protein antibodies

The correct answer is d)

Explanation:
This patient presents with a condition that obviously has significant pulmonary and
renal involvement. This is observed in the clinical presentation of hemoptysis, urine
color changes, pedal edema, and hypertension. Given the patient’s history of smoking
and recent respiratory infection, this clinical picture suggests Goodpasture syndrome.
It is a disease that causes acute glomerulonephritis and pulmonary hemorrhage due to
the presence of circulating anti-glomerular basement membrane antibodies (choice
D). The autoantibodies bind to their reactive epitopes in the basement membranes and
activate the complement cascade, resulting in tissue injury. It is a sort of type II
“hypersensitivity” reaction with antigen-antibody binding on the patient’s cell
surfaces. In almost all patients, lung injury that increases alveolar-capillary
permeability can be traced, such as the smoking history and the respiratory infection
in this patient that preceded the illness.

> Anti-double stranded DNA antibodies (choice A) are found in systemic lupus
erythematosus.
> Eosinophilia (choice B) can be seen in Churg-Strauss syndrome, a disease that some
confuse with Goodpasture syndrome because of its respiratory and renal involvement.
It is an allergic granulomatous angiitis that presents with asthma, paranasal sinusitis,
and rapidly progressive glomerulonephritis. This patient has no wheezing commonly
seen in asthma and his history is more suggestive of Goodpasture syndrome.
> Cytoplasmic antineutrophil cytoplasmic antibodies (choice C) are seen in
granulomatosis with polyangiitis formerly known as Wegener’s granulomatosis.
While the pulmonary-renal presentation of this disease in the same fashion as
Goodpasture syndrome makes it hard to distinguish the two, in Wegener’s
granulomatosis, upper airways involvement are also more common and very
significant, and ruling out nasal lesions and sinusitis should inform us that this
diagnosis is less likely than Goodpasture syndrome.
> Anti-citrullinated protein antibodies (choice E) are diagnostic of rheumatoid
arthritis.

Goodpasture syndrome presents with pulmonary hemorrhage and acute


glomerulonephritis. It is characterized by the presence of circulating anti-glomerular
basement membrane antibodies.

You might also like